[obm-l] Re: [obm-l] cadeira de 3 pés

2023-01-23 Por tôpico Eduardo Wagner
O banco de 3 pernas não balança porque nosso mundo é tridimensional.
Não tem nada a ver com plano ou triângulo. Um banco de 3 pernas não balança
se for colocado
no teto de um carro.
No mundo 2D um banco de 2 pernas não balança, mas um de 3 pernas pode
balançar.
Em um mundo 4D uma cadeira de 4 pernas não balança.
Wbs
Wagner


Em dom., 22 de jan. de 2023 às 23:24, Claudio Buffara <
claudio.buff...@gmail.com> escreveu:

> Achei na internet duas explicações distintas para a estabilidade de uma
> cadeira (ou mesa ou banco) de 3 pés.
> Aqui estão:
> https://www.somatematica.com.br/curiosidades/c98.php
>
> http://colegiofarroupilha.com.br/site/qual-cadeira-e-mais-firme-a-que-tem-tres-ou-quatro-pes/
>
> Qual das duas é a explicação correta?
> Ou nenhuma das duas? E, nesse caso, qual a explicação?
>
> []s,
> Claudio.
>
> --
> Esta mensagem foi verificada pelo sistema de antivírus e
> acredita-se estar livre de perigo.

-- 
Esta mensagem foi verificada pelo sistema de antiv�rus e
 acredita-se estar livre de perigo.



[obm-l] Re: [obm-l] Quebra do RSA por solução do problema de fatoração - Eric Campos Bastos Guedes

2022-01-11 Por tôpico Eduardo Guimarães
Chave pública:
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

Em ter., 11 de jan. de 2022 às 11:53, Eduardo Guimarães <
eduardoestudo...@gmail.com> escreveu:

>
> 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
> Desencripta isso.
>
> Em ter., 11 de jan. de 2022 às 11:19, Claudio Buffara <
> claudio.buff...@gmail.com> escreveu:
>
>> Eu diria que a melhor forma de avaliar seu trabalho é testando.
>> Apesar do “desafio RSA” já ter encerrado, os números ainda estão
>> disponíveis.
>> Da uma olhada no verbete “RSA numbers” na Wikipédia.
>>
>> Enviado do meu iPhone
>>
>> > Em 11 de jan. de 2022, à(s) 15:03, Eric Campos Bastos Guedes <
>> ebastosgue...@gmail.com> escreveu:
>> >
>> > 
>> > Proponho um algoritmo para quebrar o RSA. O algoritmo que eu propus
>> antes trabalhava com números muito grandes e por isso podia não funcionar
>> direito. Esse trabalha com números bem menores porque usa módulo N numa
>> etapa. O algoritmo e sua explicação estão no YouTube com o mesmo título
>> desse e-mail. São dois vídeos, o que conta é o mais recente deste ano de
>> 2022.
>> >
>> > QUEBRA DO RSA - ALGORITMO N.2
>> >
>> > PASSO 1: a=3
>> >
>> > inicializando o valor de a
>> >
>> > PASSO 2: N é o inteiro a ser fatoradoÂ
>> >
>> > N é o número usado no RSA. N é o produto de dois números primos
>> grandes não muito próximos.Â
>> >
>> > PASSO 3: M=N^512 (N elevado a 512)
>> >
>> > M é um número grande mas não muito grande. O valor de P não vai
>> ultrapassar muito o valor de M. P é uma variável inteira que acumula
>> fatores primos. Aí você faz MDC(P, N) para tentar fatorar N.
>> >
>> > PASSO 4: a=a+1
>> >
>> > O valor de a é atualizado para a+1, isto é,  é  acrescentado 1 ao
>> valor de a
>> >
>> > PASSO 5: P=aÂ
>> >
>> > O valor de P é inicializadoÂ
>> >
>> > PASSO 6: b = número aleatório entre 0 e 1
>> > PASSO 7: Se b > 1/2 faça c=1 senão faça c=-1
>> >
>> > O objetivo dos passos 6 e 7 é atribuir à variável c um valor que
>> pode ser 1 ou -1. Isso nem precisa ser feito de modo aleatório, mas acho
>> que vai funcionar melhor se for aleatório.Â
>> >
>> > PASSO 8: P=P(P+c)
>> >
>> > É uma atribuição de valor. O novo valor de P passa a ser P(P+c).
>> Note que P+c é relativamente primo com P. Na prática são acrescentados
>> novos fatores primos a P que vai acumular fatores primos.Â
>> >
>> > PASSO 9: Se P < M vá para o PASSO 6
>> >
>> > Esse passo determina um looping para acumular fatores em P.
>> >
>> > PASSO 10: Se MDC(P, N) for diferente de 1 vá para o PASSO 14
>> >
>> > Se MDC(P, N) for diferente de 1 ele pode ser um fator primo de N. Resta
>> verificar se ele não é o próprio N. Isso vai ser feito no PASSO 14.
>> >
>> > PASSO 11: P = Resto da divisão de P por N
>> >
>> > Esse passo é para trabalharmos com números menores.Â
>> >
>> > PASSO 12: Se P < 4 faça P=4
>> >
>> > Talvez esse passo possa ser omitido
>> >
>> > PASSO 13: vá para o PASSO 6
>> >
>> > PASSO 14: Se MDC(P, N)=N vá para o PASSO 4
>&g

[obm-l] Re: [obm-l] Quebra do RSA por solução do problema de fatoração - Eric Campos Bastos Guedes

2022-01-11 Por tôpico Eduardo Guimarães
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
Desencripta isso.

Em ter., 11 de jan. de 2022 às 11:19, Claudio Buffara <
claudio.buff...@gmail.com> escreveu:

> Eu diria que a melhor forma de avaliar seu trabalho é testando.
> Apesar do “desafio RSA” já ter encerrado, os números ainda estão
> disponíveis.
> Da uma olhada no verbete “RSA numbers” na Wikipédia.
>
> Enviado do meu iPhone
>
> > Em 11 de jan. de 2022, à(s) 15:03, Eric Campos Bastos Guedes <
> ebastosgue...@gmail.com> escreveu:
> >
> > 
> > Proponho um algoritmo para quebrar o RSA. O algoritmo que eu propus
> antes trabalhava com números muito grandes e por isso podia não funcionar
> direito. Esse trabalha com números bem menores porque usa módulo N numa
> etapa. O algoritmo e sua explicação estão no YouTube com o mesmo título
> desse e-mail. São dois vídeos, o que conta é o mais recente deste ano de
> 2022.
> >
> > QUEBRA DO RSA - ALGORITMO N.2
> >
> > PASSO 1: a=3
> >
> > inicializando o valor de a
> >
> > PASSO 2: N é o inteiro a ser fatoradoÂ
> >
> > N é o número usado no RSA. N é o produto de dois números primos
> grandes não muito próximos.Â
> >
> > PASSO 3: M=N^512 (N elevado a 512)
> >
> > M é um número grande mas não muito grande. O valor de P não vai
> ultrapassar muito o valor de M. P é uma variável inteira que acumula
> fatores primos. Aí você faz MDC(P, N) para tentar fatorar N.
> >
> > PASSO 4: a=a+1
> >
> > O valor de a é atualizado para a+1, isto é,  é  acrescentado 1 ao
> valor de a
> >
> > PASSO 5: P=aÂ
> >
> > O valor de P é inicializadoÂ
> >
> > PASSO 6: b = número aleatório entre 0 e 1
> > PASSO 7: Se b > 1/2 faça c=1 senão faça c=-1
> >
> > O objetivo dos passos 6 e 7 é atribuir à variável c um valor que pode
> ser 1 ou -1. Isso nem precisa ser feito de modo aleatório, mas acho que
> vai funcionar melhor se for aleatório.Â
> >
> > PASSO 8: P=P(P+c)
> >
> > É uma atribuição de valor. O novo valor de P passa a ser P(P+c). Note
> que P+c é relativamente primo com P. Na prática são acrescentados novos
> fatores primos a P que vai acumular fatores primos.Â
> >
> > PASSO 9: Se P < M vá para o PASSO 6
> >
> > Esse passo determina um looping para acumular fatores em P.
> >
> > PASSO 10: Se MDC(P, N) for diferente de 1 vá para o PASSO 14
> >
> > Se MDC(P, N) for diferente de 1 ele pode ser um fator primo de N. Resta
> verificar se ele não é o próprio N. Isso vai ser feito no PASSO 14.
> >
> > PASSO 11: P = Resto da divisão de P por N
> >
> > Esse passo é para trabalharmos com números menores.Â
> >
> > PASSO 12: Se P < 4 faça P=4
> >
> > Talvez esse passo possa ser omitido
> >
> > PASSO 13: vá para o PASSO 6
> >
> > PASSO 14: Se MDC(P, N)=N vá para o PASSO 4
> >
> > Se MDC(P, N) = N não foram encontrados fatores primos e algoritmo
> recomeça do ponto apropriado.Â
> >
> > PASSO 15: MDC(P, N) é fator (primo) de N
> >
> > FIM
> >
> > Eu fui menção honrosa na Olimpíada Ibero-americana de Matemática
> Universitária em 2006. Acho que este meu trabalho merece ser avaliado.
> >
> >
> >
> >
> > --
> > Esta mensagem foi verificada pelo sistema de antivírus e
> > acredita-se estar livre de perigo.
>
> --
> Esta mensagem foi verificada pelo sistema de antivírus e
>  acredita-se estar livre de perigo.
>
>
> =
> Instru�ões para entrar na lista, sair da lista e usar a lista em
> http://www.mat.puc-rio.br/~obmlistas/obm-l.html
> =
>

-- 
Esta mensagem foi verificada pelo sistema de antiv�rus e
 acredita-se estar livre de perigo.



[obm-l] Re: [obm-l] Função

2021-04-22 Por tôpico Eduardo Henrique Rodrigues do Nascimento
Cara, toda função real contínua e bijetora é monótona. Como contraexemplo
se f não for contínua:

  x+1 para x no intervalo [0,1[
f(x)={x, para x≥2 e x<0
  x-1 para x no intervalo [1,2[

então f não é crescente em todo o seu domínio: 1/2<3/2; mas
f(1/2)=3/2>1/2=f(3/2).

 além disso, a função complexa f(z)=z é claramente bijetora.

Em qui, 22 de abr de 2021 07:19, Israel Meireles Chrisostomo <
israelmchrisost...@gmail.com> escreveu:

> Me desculpem se eu estou falando bobagem, mas considere uma função com
> domínio complexo, então essa função não pode ser bijetora, pois toda função
> bijetora ou é crescente ou é decrescente, mas não há ordem nos complexos
>
> --
> Esta mensagem foi verificada pelo sistema de antivírus e
> acredita-se estar livre de perigo.

-- 
Esta mensagem foi verificada pelo sistema de antiv�rus e
 acredita-se estar livre de perigo.



Re: [obm-l] Amigo secreto ENEM

2021-01-28 Por tôpico Eduardo Guimarães
Souberam que a questão foi realmente anulada?
https://g1.globo.com/educacao/enem/2020/noticia/2021/01/27/inep-anula-duas-questoes-do-enem-2020.ghtml

Em qui., 28 de jan. de 2021 às 11:38, Arthur Queiroz 
escreveu:

> Uma pergunta: você assume que o número de sorteios é !10. Mas e se, em
> meio ao sorteio, nossa permutação caótica seja tal que seja formado um
> ciclo indesejado? Digamos A->B->C->A. Como o sorteio continuará nesse caso?
> Será escolhida aleatoriamente uma pessoa de fora do ciclo para continuar?
> Isso não afetaria esse !10?
>
> Em ter, 26 de jan de 2021 17:26, Ralph Costa Teixeira 
> escreveu:
>
>> Deixa eu copiar o que escrevi em outro lugar... :D :D
>>
>> Primeiro: não fica claro do enunciado se "auto-sorteios" (alguém sortear
>> o próprio nome) são permitidos ou não, e isto ALTERA a resposta. :(
>>
>> Vejamos possíveis respostas corretas:
>>
>> ---///---
>>
>> SE AUTO-SORTEIOS FOREM PERMITIDOS:
>> Em resumo, temos 1/10 de chance de A iniciar o sorteio, e 1/10 de chance
>> de B terminar (1/10 sim, pois A *pode* terminar). Assim, a resposta seria
>> 1/10*1/10*2=1/50.
>>
>> Com mais detalhes para justificar o segundo "1/10":
>> -- Número de sorteios possíveis = 10!
>> -- Número de sorteios que formam um único ciclo de tamanho 10 = 9!
>> -- Note que ter um ciclo de tamanho 10 equivale a terminar com quem
>> inicia; portanto, a chance de o amigo secreto terminar com o mesmo que
>> iniciou seria 9!/10!=1/10 (*que é independente de quem começa*).
>>
>> Assim:
>> -- Chance de A iniciar = 1/10;
>> Agora, DADO QUE A INICIOU:
>>  Chance de A terminar = 9!/10! = 1/10
>>  Portanto, chance de não terminar com A: 9/10
>>  Chance de B terminar (por simetria): (9/10) /9 = 1/10
>>
>> Isso nos dá 1/10*1/10 = 1/100 de chance do amigo secreto começar por A e
>> terminar com B. Portanto a resposta seria o dobro, 1/50.
>>
>> ---///---
>> SE AUTO-SORTEIOS SÃO PROIBIDOS:
>> -- Número de sorteios (desarranjos) possíveis = !10 (vou escrever K=!10
>> daqui por diante);
>> -- Número de sorteios que formam um único ciclo de tamanho 10 = 9!
>> -- Portanto, a chance de o amigo secreto terminar com o mesmo que iniciou
>> seria 9!/K (que é independente de quem começa).
>>
>> Assim:
>> -- Chance de A iniciar = 1/10;
>> Agora, DADO QUE A INICIOU:
>>  Chance de A terminar = 9!/K
>>  Portanto, chance de não terminar com A: 1-9!/K
>>  Chance de B terminar (por simetria): (1-9!/K) /9 = (K-9!)/(9K)
>>
>> Isso nos dá 1/10* (K-9!)/(9K) = (K-9!)/(90K) de chance do amigo secreto
>> começar por A e terminar com B. Portanto a resposta seria o dobro,
>> (K-9!)/(45K). Fazendo a conta com a ajuda do computador, achei 12001/741645.
>>
>> Abraço, Ralph.
>>
>>
>>
>> On Tue, Jan 26, 2021 at 1:45 PM Professor Vanderlei Nemitz <
>> vanderma...@gmail.com> wrote:
>>
>>> Oi, pessoal!
>>>
>>> Com certeza vocês estão acompanhando desde domingo as resoluções da
>>> questão do ENEM do amigo secreto.
>>> Além da resposta proposta, *1/45*, que parece não estar correta, já vi
>>> outras duas, *12001/741645* (ETAPA e ANGLO), que consideram também que
>>> o sorteio anterior para definir "quem presenteia quem", e *7/360*, do
>>> vídeo a seguir:
>>>
>>> https://www.youtube.com/watch?v=c-t_BAMASKE
>>>
>>> Gostaria da opinião (e se possível, uma resolução) dos especialistas da
>>> lista (Ralph e cia :))
>>>
>>> Muito obrigado!
>>>
>>>
>>>
>>>
>>>
>>


[obm-l] Desigualdade isoperimétrica

2019-08-29 Por tôpico Eduardo Henrique
Olá pessoal, tudo bem?

Alguém tem o artigo sobre desigualdade isoperimétrica que saiu na revista 
matemática universitária em pdf para me enviar?

O link no site deles está fora...

Att.

Eduardo



-- 
Esta mensagem foi verificada pelo sistema de antivírus e
 acredita-se estar livre de perigo.



[obm-l] Somatórios

2019-07-20 Por tôpico Eduardo Henrique
Pessoal, podem me indicar algum material que explique como funcionam os 
somatórios? Gostaria de algum que explicasse em que casos podemos inverter 
somatórios, quais as condições... tanto pra finitos quanto pra infinitos. Pode 
ser apenas nomes de livros que tenham isso que eu corro atrás. Valeu!


-- 
Esta mensagem foi verificada pelo sistema de antivírus e
 acredita-se estar livre de perigo.



[obm-l] RE: Rodada de Negócios

2019-03-28 Por tôpico Eduardo Henrique
Ah, outra coisa. Parece-me que podem haver configurações diferentes das que eu 
coloquei no email anterior:

Coisas como: Se tem 23 pessoas, podemos ter

4 mesas de 5 e uma de 3
5 mesas de 4 e uma de 3
6 mesas de 3 e uma de 5 e aí por diante.

É um problema que eu não saio do lugar... Qualquer indicação de leitura é bem 
vinda, ou até mesmo o esboço de um raciocínio para que eu possa pensar em cima 
seria ótimo!

De: Eduardo Henrique
Enviado: quinta-feira, 28 de março de 2019 15:28
Para: obm-l@mat.puc-rio.br
Assunto: Rodada de Negócios

Olá pessoal, tudo bem?

Minha mãe está estudando RH e ontem ela participou de uma dinâmica que chamaram 
de 'rodada de negócios'.

Consiste no seguinte:

É uma reunião normalmente de empresários ou freelancers, distribuídos em várias 
mesas e cada um tem algum tempo para se apresentarem. Após todos se 
apresentarem em sua primeira mesa, todos trocam de mesa de maneira que não haja 
pessoas repetidas. A rodada acaba quando todos se apresentam a todos.

Por exemplo, se tivermos 9 integrantes: A, B, C, D, E, F, G, H, I, dispostos em 
3 mesas com 3 integrantes cada.

Rodada 1:
Mesa: A, B, C
Mesa: D, E, F
Mesa: G, H, I

Rodada 2:
Mesa: A, D, G
Mesa: B, E, H
Mesa: C, F, I

Rodada 3:
Mesa: A, E, I
Mesa: B, F, G
Mesa: C, D, H

Rodada 4:
Mesa: A, F, H
Mesa: B, D, I
Mesa: C, E, G

Daí eu achei interessante a proposta e resolvi dar uma estudada em como 
generalizar o conceito pra mais pessoas. Algumas perguntas que me surgiram: Se 
eu tiver 60 pessoas, 6 mesas de 10 pessoas dá certo de fazer essa rodada de 
negócios? e 10 mesas de 6? Gostaria de tentar entender como trabalhar com esse 
problema, se achar condições para saber como distribuir N pessoas em M mesas de 
forma que a rodada 'funcione' sem repetições. Vocês tem alguma dica de algum 
material para eu estudar sobre isso?


-- 
Esta mensagem foi verificada pelo sistema de antivírus e
 acredita-se estar livre de perigo.



[obm-l] Rodada de Negócios

2019-03-28 Por tôpico Eduardo Henrique
Olá pessoal, tudo bem?

Minha mãe está estudando RH e ontem ela participou de uma dinâmica que chamaram 
de 'rodada de negócios'.

Consiste no seguinte:

É uma reunião normalmente de empresários ou freelancers, distribuídos em várias 
mesas e cada um tem algum tempo para se apresentarem. Após todos se 
apresentarem em sua primeira mesa, todos trocam de mesa de maneira que não haja 
pessoas repetidas. A rodada acaba quando todos se apresentam a todos.

Por exemplo, se tivermos 9 integrantes: A, B, C, D, E, F, G, H, I, dispostos em 
3 mesas com 3 integrantes cada.

Rodada 1:
Mesa: A, B, C
Mesa: D, E, F
Mesa: G, H, I

Rodada 2:
Mesa: A, D, G
Mesa: B, E, H
Mesa: C, F, I

Rodada 3:
Mesa: A, E, I
Mesa: B, F, G
Mesa: C, D, H

Rodada 4:
Mesa: A, F, H
Mesa: B, D, I
Mesa: C, E, G

Daí eu achei interessante a proposta e resolvi dar uma estudada em como 
generalizar o conceito pra mais pessoas. Algumas perguntas que me surgiram: Se 
eu tiver 60 pessoas, 6 mesas de 10 pessoas dá certo de fazer essa rodada de 
negócios? e 10 mesas de 6? Gostaria de tentar entender como trabalhar com esse 
problema, se achar condições para saber como distribuir N pessoas em M mesas de 
forma que a rodada 'funcione' sem repetições. Vocês tem alguma dica de algum 
material para eu estudar sobre isso?


-- 
Esta mensagem foi verificada pelo sistema de antivírus e
 acredita-se estar livre de perigo.



Re: [obm-l] Algo errado com o enunciado?

2019-03-18 Por tôpico Eduardo Wagner
Prezado Pedro:

Relaxe. Não há nenhum conjunto obrigatório para os naturais. Cada um adota
o que quiser, com o zero ou sem o zero.
Em sequências costuma-se adotar o conjunto dos naturais sem o zero, pois
quando estamos contando elementos
de algum conjunto, a maioria das pessoas normais não começa a contar pelo
zero.
Em aritmética o zero deve ser incluído como elemento neutro da adição.
Oberve que nos axiomas de Peano, o zero não está incluído. Isso é
conveniente pois o axioma 4 é a base para o
Princípio da indução.
Enfim, essas questões de nomenclatura não são importantes.
Adote a que for mais conveniente ao objeto que você está estudando.
Att,
E, Wagner.

Em sáb, 16 de mar de 2019 às 14:41, Pedro José 
escreveu:

> Boa tarde!
>
> Já questionei uma vez aqui no sítio sobre um fato, para mim curioso.
> Estudara no ginásio e também no científico que os inteiros positivos,
> representado por um Z estilizado e um sinal de adição eram elementos do
> conjunto {0, 1, 2, 3,...} e os inteiros estritamente positivos teriam a
> representação por um Z estilizado um sinal de adição e um asterisco e
> seriam elementos de {1, 2, 3, 4, 5,...} Então havia interseção entre os
> conjuntos dos inteiros positivos e negativos que seria obviamente o {0}. O
> mesmo acontecia com os reais positivos.
> Ainda relembro Miguel Jorge e Dona Frida chamando a atenção entre positivo
> e estritamente positivo.
> Futuramente deparei-me com esse novo conceito.
> Outra coisa Miguel Jorge costumava começar seus livros com o capítulo 0
> para frisar que ele considerava zero natural, hoje não encontro uma posição
> pacífica, já vi livros onde o zero não é considerado natural. Todavia,
> nunca mais vi quem considere zero um interior positivo.
> Gostaria de saber a razão da mudança. Se a corrente que estudei era uma
> dissidência que não pegou ou se de fato ocorreu alguma mudança para nos
> adequarmos a um entendimento mais global??
>
> Saudações,
> PJMS
>
>
>
>
>
>
>
> Em qui, 7 de mar de 2019 às 06:10, Anderson Torres <
> torres.anderson...@gmail.com> escreveu:
>
>> Em qua, 6 de mar de 2019 às 16:41, marcone augusto araújo borges
>>  escreveu:
>> >
>> > Seja f uma função definida para todo inteiro positivo tal que
>> >
>> > i) f(0) = 1
>> > ii) f(2n + 1) =  2f(n) + 1
>> > iii) f(2n) = 3f(n)
>> > .
>> > .
>> > .
>> >
>> > se vale para todo inteiro POSITIVO, porque começa com f(0)?
>>
>> Qual é a origem do problema?
>> Talvez tenha sido um mero ato-falho do examinador. Afinal, não me
>> parece que o problema prossegue insolúvel se supusermos "naturais" em
>> vez de "inteiros positivos".
>>
>> >
>> > --
>> > Esta mensagem foi verificada pelo sistema de antivírus e
>> > acredita-se estar livre de perigo.
>>
>> --
>> Esta mensagem foi verificada pelo sistema de antivírus e
>>  acredita-se estar livre de perigo.
>>
>>
>> =
>> Instru�ões para entrar na lista, sair da lista e usar a lista em
>> http://www.mat.puc-rio.br/~obmlistas/obm-l.html
>> =
>>
>
> --
> Esta mensagem foi verificada pelo sistema de antivírus e
> acredita-se estar livre de perigo.

-- 
Esta mensagem foi verificada pelo sistema de antiv�rus e
 acredita-se estar livre de perigo.



[obm-l] Re: [obm-l] Re: [obm-l] TRIÂNGULO

2019-03-11 Por tôpico Eduardo Wagner
Analítica. Adote AE como unidade de comprimento.
Resp: PQ/QR = 7/5

Em sáb, 9 de mar de 2019 às 12:40, Anderson Torres <
torres.anderson...@gmail.com> escreveu:

>
>
>
> Em qui, 7 de mar de 2019 às 07:47, Vanderlei Nemitz 
> escreveu:
>
>> Só enxerguei uma saída usando geometria analítica. Alguma ideia?
>> Muito obrigado!
>>
>> *Dado um triângulo ABC, com Â= 90º, D é o ponto médio de BC, F é o ponto
>> médio de AB, E é o ponto médio de AF e G o ponto médio de FB. AD intersecta
>> CE, CF, CG em P, Q e R respectivamente. Determine a razão PQ/QR.*
>>
>>
> A ideia que pensei foi usar Razão Cruzada.
>
> https://www.cut-the-knot.org/pythagoras/Cross-Ratio.shtml
> Mas só isso não vai adiantar.
>
>
>
>>
>>
>>
>>
>> 
>>  Livre
>> de vírus. www.avast.com
>> .
>>
>> <#m_5191055488509645045_m_3774298393707173559_m_6555290746475537769_DAB4FAD8-2DD7-40BB-A1B8-4E2AA1F9FDF2>
>>
>> --
>> Esta mensagem foi verificada pelo sistema de antivírus e
>> acredita-se estar livre de perigo.
>
>
> --
> Esta mensagem foi verificada pelo sistema de antivírus e
> acredita-se estar livre de perigo.

-- 
Esta mensagem foi verificada pelo sistema de antiv�rus e
 acredita-se estar livre de perigo.



[obm-l] Re: [obm-l] Dúvida conceitual (equações)

2018-10-16 Por tôpico Eduardo Wagner
Considere multiplicidades.

Em dom, 14 de out de 2018 às 06:38, Vanderlei Nemitz 
escreveu:

> Bom dia!
> Na seguinte questão, que me foi apresentada por um aluno, a resposta
> proposta é a alternativa C (1/2). Eu sempre pensei que apenas
> considerávamos multiplicidades em equações polinomiais. Como essa é uma
> equação exponencial, obtive a resposta B (-1/2). O que é correto pensar?
>
> O produto das raízes da equação 16.4^3x - 40.4^2x + 17.4^x - 2 = 0 é igual
> a:
> A) 1
> B) - 0,5
> C) 0,5
> D) - 1
> E) 0
>
> Muito obrigado!
>
> --
> Esta mensagem foi verificada pelo sistema de antivírus e
> acredita-se estar livre de perigo.

-- 
Esta mensagem foi verificada pelo sistema de antiv�rus e
 acredita-se estar livre de perigo.



[obm-l] Quadilátero

2018-09-06 Por tôpico Eduardo Wilner
a) Construir um quadrilátero convexo cujos lados opostos sejam diâmetros de 
circunferências que se tangenciam,     duas à duas, no interior do polígono.b) 
Calcular a distância entre os pontos de tangencia, em função do perímetro.   

-- 
Esta mensagem foi verificada pelo sistema de antiv�rus e
 acredita-se estar livre de perigo.



[obm-l] Constução geométrica livre

2018-01-29 Por tôpico Eduardo Wilner
Construir um triangulo isoceles ABC de base AC=2sqrt[6], cuja 
ceviana AD forma um ângulo de 15° com a base, sendo D um ponto 
do lado BC com BD=2.

-- 
Esta mensagem foi verificada pelo sistema de antiv�rus e
 acredita-se estar livre de perigo.



[obm-l] Dúvida combinatória

2017-11-13 Por tôpico Eduardo Henrique
Pessoal, estava estudando o seguinte tipo de problema:

Quantas são as soluções inteiras positivas de a+b+c=r, com r inteiro positivo. 
Até aqui ok. A dúvida veio depois:

Quantas são as solução inteiras positivas de 1a+2b+3c=r? E mais geralmente, de 
1k_1+...+n_kn=r? Alguém sabe como abordar esse tipo de problema ou então 
saberia me indicar um material de estudos?

Obrigado.

Eduardo

-- 
Esta mensagem foi verificada pelo sistema de antivírus e
 acredita-se estar livre de perigo.



[obm-l] solução de questões da OBM

2017-11-09 Por tôpico Eduardo Wilner
Alguém sabe como se pode acessar solução (gabarito...) da 3ª fase (Nivel 2) da 
OBM, de 2015 por exemplo?Obrigado.


-- 
Esta mensagem foi verificada pelo sistema de antiv�rus e
 acredita-se estar livre de perigo.



[obm-l] Re: Questão de vetores

2016-09-18 Por tôpico Eduardo Wilner
Não entrou na minha caixa postal (e-mail)  e tbm não conseguí responder 
diretamente da lista. Vejamos se assim "funciona"...
 Sejam BM=x; MC=y, logo 3x=5y ou y=x*3/5 e BC=x+y=x*8/5 logo x=BM=BC*5/8. 

-- 
Esta mensagem foi verificada pelo sistema de antiv�rus e
 acredita-se estar livre de perigo.



[obm-l] Retângulo com "cúpula".

2016-09-17 Por tôpico Eduardo Wilner
ABCD é um retângulo tal que BC=AB.cos 45° e P é um ponto da 
semi-circunferência de diâmetro AB, externo ao retângulo e mais
próximo à B; sejam K e L as intersecções de AB com PD e PC, 
respectivamente.
  Q é outro ponto da semi-circunferência (vizinho à P) tal que 
AQ=AL. Se os prolongamentos de QK e QL encontram, respectivamente, 
CD em G e CB em H, provar que GAQH é um paralelograma.

-- 
Esta mensagem foi verificada pelo sistema de antiv�rus e
 acredita-se estar livre de perigo.



[obm-l] Integração

2016-02-02 Por tôpico Eduardo Henrique
Pessoal, alguém saberia me dizer quando vale o seguinte:

int a até b de f(x)dx = int -b até -a de f(-x)dx ? 
Att.,
Eduardo


Este e-mail foi enviado por um computador sem vírus e protegido 
pelo Avast. www.avast.com   


  

[obm-l] RE: [obm-l] Soma de números compostos

2015-12-12 Por tôpico Eduardo Henrique
Cara, acho que todo natural ímpar maior que 11 se escreve como 9+2*n, n natural.

Att., 
Eduardo

From: marconeborge...@hotmail.com
To: obm-l@mat.puc-rio.br
Subject: [obm-l] Soma de números compostos
Date: Sat, 12 Dec 2015 01:36:39 +




Mostre que todo inteiro n > 11 pode ser escrito como soma de númeroscompostos 
positivos
para n par : n = 11 + 2t-1 = 4 + [2(t + 3)]mas...   
  
--

Esta mensagem foi verificada pelo sistema de antivírus e 

 acredita-se estar livre de perigo.


  
-- 
Esta mensagem foi verificada pelo sistema de antivírus e
 acredita-se estar livre de perigo.



[obm-l] Matriz nxn

2015-11-03 Por tôpico Eduardo Henrique
Pessoas, me deparei com a seguinte questão:

Seja M uma matriz nxn com x na diagonal principal, e a>0 nas demais posições. 
Calcule det(M).

Alguém poderia me indicar um caminho para seguir? Eu não consegui avançar nada 
nessa questão :(
Att.
Eduardo   
-- 
Esta mensagem foi verificada pelo sistema de antivírus e
 acredita-se estar livre de perigo.



RE: [obm-l] Derivada de e^z

2015-09-10 Por tôpico Eduardo Henrique
Ah, z é complexo. Jurava ter escrito isso, desculpe. Sim, pela definição de 
derivada: lim_{h\rightarrow0}[f(z+h)-f(z)]/h

Date: Thu, 10 Sep 2015 16:59:44 -0700
From: regisgbar...@yahoo.com.br
Subject: Re: [obm-l] Derivada de e^z
To: obm-l@mat.puc-rio.br


Pela definição da Derivada?  E z é um número real ou complexo? Regis
Enviado do Yahoo Mail no Android
 De:"Eduardo Henrique" <dr.dhe...@outlook.com>
Data:18:54 Qui, 10 de Set de PM
Assunto:[obm-l] Derivada de e^z

 Pessoal, to batendo a cabeça aqui faz uns dois dias e não sai. Tem como provar 
pela definição que a derivada de e^z é e^z?
Att.
Eduardo   
--

Esta mensagem foi verificada pelo sistema de antivírus e 

 acredita-se estar livre de perigo.

--

Esta mensagem foi verificada pelo sistema de antivírus e 

 acredita-se estar livre de perigo.   
-- 
Esta mensagem foi verificada pelo sistema de antivírus e
 acredita-se estar livre de perigo.



[obm-l] Derivada de e^z

2015-09-10 Por tôpico Eduardo Henrique
Pessoal, to batendo a cabeça aqui faz uns dois dias e não sai. Tem como provar 
pela definição que a derivada de e^z é e^z?
Att.
Eduardo   
-- 
Esta mensagem foi verificada pelo sistema de antivírus e
 acredita-se estar livre de perigo.



[obm-l] Determinante máximo

2015-08-18 Por tôpico Eduardo Henrique
Amigos, alguma ideia de como resolver isso:

Se tem uma matriz 4x4 com 8 0's, 4 1's e 4 -1's, qual o maior valor pro 
determinante dela?
Att.
Eduardo   
-- 
Esta mensagem foi verificada pelo sistema de antivírus e
 acredita-se estar livre de perigo.



[obm-l] Bases em R^3

2015-07-05 Por tôpico Eduardo Henrique
Pessoal, uma questão aqui dum livro, e eu não consegui fazer:
Seja A o conjunto de todos os vetores em R³ cujas coordenadas são 0's ou 1's.
Quantas bases de R³ podemos formar com elementos desse 
conjunto?__Inicialmente eu pensei assim:
Esse conjunto contém 8 elementos, porém o (0,0,0) não ajudará na formação duma 
base, logo pro primeiro elemento da base eu teria 7 possibilidades, pra segunda 
6 possibilidades e pra última, apenas 5. Como temos 3 elementos da base, há 6 
permutações desses elementos. Então achei que eram 35 (7x5) elementos... Depois 
vi que eu considerei os vetores (1,1,1),(1,1,0),(0,0,1) que não formam uma 
base...
Como resolver isso? Poderiam me ajudar?
Att.
Eduardo   
-- 
Esta mensagem foi verificada pelo sistema de antivírus e
 acredita-se estar livre de perigo.



[obm-l] Dúvida 'boba' de contagem

2015-06-04 Por tôpico Eduardo Henrique
E ai pessoas, tudo bem?
Tava resolvendo uns exercícios sobre contagem duma apostila aqui e me deparei 
com este exercício:
Num Ka Kay, o oriental famoso por sua inabalável paciência, deseja bater o 
recorde mundial de construção de castelo de cartas.Ele vai montar um castelo na 
forma de um prisma triangular no qual cada par de cartas inclinadas que se 
tocam deve estar apoiado em uma carta horizontal, excetuando-se as cartas da 
base, que estão apoiadas em uma mesa.
Num Ka Kay quer construir um castelo com 40 níveis. Determine o número de 
cartas que ele vai utilizar.
__Eu sei resolver essa questão por PA, ms está na 
parte de combinatória, em particular, na parte de princípio fundamental da 
contagem (se uma decisão d1 pode ser tomada de m maneiras e uma vez tomada a 
decisão d1, uma decisão d2 pode ser tomada de n maneiras, então o número de 
formas de se decidir d1 e depois d2 é m.n). Alguém poderia me ajudar nessa 
utilizando esse princípio?
Att.
Eduardo   
-- 
Esta mensagem foi verificada pelo sistema de antivírus e
 acredita-se estar livre de perigo.



[obm-l] Área da Cicloide

2015-05-24 Por tôpico Eduardo Henrique
Eu lendo um livro de história da matemática vi que Torricelli e Wren 
conseguiram demonstrar que a área sob um arco de cicloide é 3x a área do 
circulo que a gera utilizando o método da exaustão! Alguém saberia me indicar 
onde conseguir essas demonstrações ou até mesmo me dar uma luz em como faze-la?
Att
Eduardo   
-- 
Esta mensagem foi verificada pelo sistema de antivírus e
 acredita-se estar livre de perigo.



[obm-l] RE: [obm-l] Potência de sete

2015-05-22 Por tôpico Eduardo Henrique
Posso estar errado, mas você não pode tomar 10^(n+1)  7^k  10^(n+2) e 
inverter? Dai você teria 10^(-n-2) 7^-k  10^(-n-1)

O primeiro número [10^(-n-2)] tem n-1 zeros, enquanto o último [10^(-n-1)] 
teria n zeros, como 7^(-k) está entre eles... Eu concluiria que o problema está 
resolvido. 
Como já disse, posso estar errado.
Att.
Eduardo

From: marconeborge...@hotmail.com
To: obm-l@mat.puc-rio.br
Subject: [obm-l] Potência de sete
Date: Fri, 22 May 2015 20:41:42 +




Mostre que, para todo inteiro positivo n, existe uma potência de sete cuja 
representação decimalcontem pelo menos n zeros sucessivos   
   
--

Esta mensagem foi verificada pelo sistema de antivírus e 

 acredita-se estar livre de perigo.   
-- 
Esta mensagem foi verificada pelo sistema de antivírus e
 acredita-se estar livre de perigo.



[obm-l] Área da Ciclóide

2015-05-08 Por tôpico Eduardo Henrique
Pessoal, uma dúvida me surgiu. Há alguma forma de determinar a área de uma 
ciclóide sem ser por meio de integração? Estava pensando em algo como método da 
alavanca de arquimedes ou princípio de cavalieri. Alguém sabe alguma?
Att.
Eduardo   
-- 
Esta mensagem foi verificada pelo sistema de antivírus e
 acredita-se estar livre de perigo.



RE: [obm-l] Primos consecutivos

2015-04-13 Por tôpico Eduardo Henrique
Tente mostrar que para cada sequência de 3 números naturais da forma: N, N+2, 
N+4, pelo menos um deles é múltiplo de 3.
Att.
Eduardo

 From: brped...@hotmail.com
 To: obm-l@mat.puc-rio.br
 Subject: [obm-l] Primos consecutivos
 Date: Tue, 14 Apr 2015 04:48:14 +0300
 
 Caros Colegas,
 
 Sabendo que a, a + 2 e a + 4 são números primos, como provar que a = 3?
 
 (Números primos são os inteiros que têm exatamente 4 divisores.)
 
 Abraços!
 Pedro Chaves
 -- 
 Esta mensagem foi verificada pelo sistema de antivírus e
  acredita-se estar livre de perigo.
 
 
 =
 Instruções para entrar na lista, sair da lista e usar a lista em
 http://www.mat.puc-rio.br/~obmlistas/obm-l.html
 =
  
-- 
Esta mensagem foi verificada pelo sistema de antivírus e
 acredita-se estar livre de perigo.



RE: [obm-l] Como provar?

2015-03-27 Por tôpico Eduardo Henrique
Ah, somatório de 2 elevado a i, com indice i nos naturais. Na verdade eu 
escrevi menos do que eu deveria, pois na verdade temos que é um somatório de 
alpha sub-indice i vezes 2^i, o índice i pertencente aos naturais.

From: marconeborge...@hotmail.com
To: obm-l@mat.puc-rio.br
Subject: [obm-l] Como provar?
Date: Fri, 27 Mar 2015 12:23:41 +




Eduardo, o que significa sum_ i 2^i ? 
--

Esta mensagem foi verificada pelo sistema de antivírus e 

 acredita-se estar livre de perigo.   
-- 
Esta mensagem foi verificada pelo sistema de antivírus e
 acredita-se estar livre de perigo.



RE: [obm-l] Como provar?

2015-03-26 Por tôpico Eduardo Henrique
Cara, pro 1) eu posso estar muito errado, mas não sai por indução?

Digo, 1= 2^0
2=2^1

supomos que n = sum_i 2^i

para n+1 temos n+1 =sum_i 2^i +1 = sum_ i^k 2^i + 2^0. Dai você argumenta um 
pouquinho que essa soma é da forma que tu quer. 
Será que falei muita besteira? 
Abraços
Eduardo

From: marconeborge...@hotmail.com
To: obm-l@mat.puc-rio.br
Subject: [obm-l] Como provar?
Date: Fri, 27 Mar 2015 00:15:46 +




1) Prove que todo número natural pode ser representado como soma de diversas 
potências distintas de base 2
2) Prove que qualquer número natural pode ser representado como a soma de 
diversos números de Fibonacci diferentes
--

Esta mensagem foi verificada pelo sistema de antivírus e 

 acredita-se estar livre de perigo.   
-- 
Esta mensagem foi verificada pelo sistema de antivírus e
 acredita-se estar livre de perigo.



[obm-l] RE: [obm-l] RE: [obm-l] RE: [obm-l] Dúvidas

2015-03-06 Por tôpico Eduardo Henrique
Leia melhor a pergunta, e verifique que minha resposta é condizente. Note que 
em nenhum momento eu estou limitado a um grupo de 40 pessoas e no enunciado não 
fala que eu devo 'substituir' os homens por mulheres.

Att.

Eduardo

From: claudiot...@hotmail.com
To: obm-l@mat.puc-rio.br
Subject: [obm-l] RE: [obm-l] RE: [obm-l] Dúvidas
Date: Fri, 6 Mar 2015 13:13:06 +




Mas, como 100 mulheres devem ser retiradas , se no grupo tem no máximo 40 
pessoas??
Eu fiz deu 20 mulheres , não sei se estou errado.
Abraços

From: dr.dhe...@outlook.com
To: obm-l@mat.puc-rio.br
Subject: [obm-l] RE: [obm-l] Dúvidas
Date: Fri, 27 Feb 2015 05:28:16 +0300




Num grupo de 40 adultos, exatamente 30% são de sexo feminino. Há várias 
maneiras de se aumentar essa porcentagem, seja introduzindo mulheres no grupo 
,seja excluindo homens dele.

 a)Quantas mulheres devem ser introduzidas no grupo ,de modo que a porcentagem 
de mulheres passe para 80%? Há 40 pessoas no grupo, 30% mulheres, logo há 12 
mulheres e portanto 28 homens. Queremos 'colocar' mulheres para que o 
percentual chegue em 80%, dai então os 28 homens serão apenas 20% de um novo 
grupo maior. este grupo maior terá 140 pessoas (por que?). Logo você tem 140-28 
= 112 mulheres nesse novo grupo, mas 12 já estavam lá, logo introduziu 100 
mulheres.
 

b)Quantos homens devem ser excluídos do grupo, de modo que a porcentagem de 
mulheres passe para 80%?Você quer que 12 mulheres sejam 80% do novo grupo;
12 - 80%
x - 100% = x=15. Ora, você tinha 28 homens no grupo, sobraram 15, logo você 
retirou 13.

  
--

Esta mensagem foi verificada pelo sistema de antivírus e 

 acredita-se estar livre de perigo.   
--

Esta mensagem foi verificada pelo sistema de antivírus e 

 acredita-se estar livre de perigo.   
--

Esta mensagem foi verificada pelo sistema de antivírus e 

 acredita-se estar livre de perigo.   
-- 
Esta mensagem foi verificada pelo sistema de antivírus e
 acredita-se estar livre de perigo.



[obm-l] RE: [obm-l] Dúvida

2015-03-05 Por tôpico Eduardo Henrique
A menos de contas erradas, creio dar 25% de moças da turma. 

Utilizando o procedimento que eu sugeri ali no outro email, R=1-M = 
0,95M+0,75(1-M)=0,8 = 0,95M - 0,75M +0,75 = 0,8 =

= 0,2M=0,05 = M=0,25 = 25%.

From: dr.dhe...@outlook.com
To: obm-l@mat.puc-rio.br
Subject: RE: [obm-l] Dúvida
Date: Fri, 6 Mar 2015 05:05:17 +0300




Rapazes+Moças = R+M = 1 = 100%
3R/4 + 95M/100 = 0,8

Sistema de duas eq. e duas incógnitas. Só resolver isolando R na primeira eq e 
substituindo na segunda.

Att.

Eduardo

From: claudiot...@hotmail.com
To: obm-l@mat.puc-rio.br
Subject: [obm-l] Dúvida
Date: Fri, 6 Mar 2015 00:41:50 +




Numa certa
turma, praticam algum esporte, exatamente, 3 em cada 4 rapazes e, também, 95%
das meninas. No total isso representa 80 % das pessoas desta turma. Qual é a 
porcentagem
de moças nesta turma?




Alguém tem uma solução rápida para esta questão.

Agradeço

From: zitinho...@hotmail.com
To: obm-l@mat.puc-rio.br
Subject: [obm-l] RE: [obm-l] Dúvidas
Date: Sun, 1 Mar 2015 14:43:10 -0300




primeiro veja que temos 28 homens e 12 mulheres.
resp a)
para que a porcentagem de mulheres passe para 80% a de homens tem que ser 
20%.28 está para 20%, assim como x está pra 80%. Sendo x o nº total de mulheres 
no grupo.fazendo a regrinha de três encontramos x = 112 mulheres no novo grupo. 
logo 112 - 12 = 100 mulheres adicionadas
resp b)seguindo um raciocínio parecido teremos que12 está para 80% assim como y 
está para 20. Sendo y o nº total de homens no grupo.fazendo a regrinha de três 
encontramos y = 3 homens no grupo. Logo 28 - 3 = 25 homens excluídos do grupo.


Num grupo de 40 adultos, exatamente 30% são de sexo feminino. Há várias 
maneiras de se aumentar essa porcentagem, seja introduzindo mulheres no grupo 
,seja excluindo homens dele.
a)Quantas mulheres devem ser introduzidas no grupo ,de modo que a porcentagem 
de mulheres passe para 80%? 
b)Quantos homens devem ser excluídos do grupo, de modo que a porcentagem de 
mulheres passe para 80%?

  
--

Esta mensagem foi verificada pelo sistema de antivírus e 

 acredita-se estar livre de perigo.   
--

Esta mensagem foi verificada pelo sistema de antivírus e 

 acredita-se estar livre de perigo.   
--

Esta mensagem foi verificada pelo sistema de antivírus e 

 acredita-se estar livre de perigo. 
  
-- 
Esta mensagem foi verificada pelo sistema de antivírus e
 acredita-se estar livre de perigo.



[obm-l] RE: [obm-l] Dúvida

2015-03-05 Por tôpico Eduardo Henrique
Rapazes+Moças = R+M = 1 = 100%
3R/4 + 95M/100 = 0,8

Sistema de duas eq. e duas incógnitas. Só resolver isolando R na primeira eq e 
substituindo na segunda.

Att.

Eduardo

From: claudiot...@hotmail.com
To: obm-l@mat.puc-rio.br
Subject: [obm-l] Dúvida
Date: Fri, 6 Mar 2015 00:41:50 +




Numa certa
turma, praticam algum esporte, exatamente, 3 em cada 4 rapazes e, também, 95%
das meninas. No total isso representa 80 % das pessoas desta turma. Qual é a 
porcentagem
de moças nesta turma?




Alguém tem uma solução rápida para esta questão.

Agradeço

From: zitinho...@hotmail.com
To: obm-l@mat.puc-rio.br
Subject: [obm-l] RE: [obm-l] Dúvidas
Date: Sun, 1 Mar 2015 14:43:10 -0300




primeiro veja que temos 28 homens e 12 mulheres.
resp a)
para que a porcentagem de mulheres passe para 80% a de homens tem que ser 
20%.28 está para 20%, assim como x está pra 80%. Sendo x o nº total de mulheres 
no grupo.fazendo a regrinha de três encontramos x = 112 mulheres no novo grupo. 
logo 112 - 12 = 100 mulheres adicionadas
resp b)seguindo um raciocínio parecido teremos que12 está para 80% assim como y 
está para 20. Sendo y o nº total de homens no grupo.fazendo a regrinha de três 
encontramos y = 3 homens no grupo. Logo 28 - 3 = 25 homens excluídos do grupo.


Num grupo de 40 adultos, exatamente 30% são de sexo feminino. Há várias 
maneiras de se aumentar essa porcentagem, seja introduzindo mulheres no grupo 
,seja excluindo homens dele.
a)Quantas mulheres devem ser introduzidas no grupo ,de modo que a porcentagem 
de mulheres passe para 80%? 
b)Quantos homens devem ser excluídos do grupo, de modo que a porcentagem de 
mulheres passe para 80%?

  
--

Esta mensagem foi verificada pelo sistema de antivírus e 

 acredita-se estar livre de perigo.   
--

Esta mensagem foi verificada pelo sistema de antivírus e 

 acredita-se estar livre de perigo.   
--

Esta mensagem foi verificada pelo sistema de antivírus e 

 acredita-se estar livre de perigo.   
-- 
Esta mensagem foi verificada pelo sistema de antivírus e
 acredita-se estar livre de perigo.



[obm-l] RE: [obm-l] Dúvidas

2015-02-26 Por tôpico Eduardo Henrique
Ops, na última conta ali eu cometi um erro, me perdoe.

x=15 implica que o novo grupo tem 15 pessoas e como 12 são mulheres, sobraram 
apenas 3 homens, logo 25 homens foram retirados.

From: dr.dhe...@outlook.com
To: obm-l@mat.puc-rio.br
Subject: RE: [obm-l] Dúvidas
Date: Fri, 27 Feb 2015 05:28:16 +0300




Num grupo de 40 adultos, exatamente 30% são de sexo feminino. Há várias 
maneiras de se aumentar essa porcentagem, seja introduzindo mulheres no grupo 
,seja excluindo homens dele.

 a)Quantas mulheres devem ser introduzidas no grupo ,de modo que a porcentagem 
de mulheres passe para 80%? Há 40 pessoas no grupo, 30% mulheres, logo há 12 
mulheres e portanto 28 homens. Queremos 'colocar' mulheres para que o 
percentual chegue em 80%, dai então os 28 homens serão apenas 20% de um novo 
grupo maior. este grupo maior terá 140 pessoas (por que?). Logo você tem 140-28 
= 112 mulheres nesse novo grupo, mas 12 já estavam lá, logo introduziu 100 
mulheres.
 

b)Quantos homens devem ser excluídos do grupo, de modo que a porcentagem de 
mulheres passe para 80%?Você quer que 12 mulheres sejam 80% do novo grupo;
12 - 80%
x - 100% = x=15. Ora, você tinha 28 homens no grupo, sobraram 15, logo você 
retirou 13.

  
--

Esta mensagem foi verificada pelo sistema de antivírus e 

 acredita-se estar livre de perigo. 
  
-- 
Esta mensagem foi verificada pelo sistema de antivírus e
 acredita-se estar livre de perigo.



RE: [obm-l] Provar que...

2014-12-19 Por tôpico Eduardo Henrique
Tenta reagrupar 100!, talvez algo como (1*100)(2*99)(3*98)...(50*51), dai você 
terá 50 produtos, cada um deles é equiparável a 50² (a saber menor), dai tem 
que argumentar um pouquinho, mas acho que sai.
AbraçosEdu

From: maikinho0...@hotmail.com
To: obm-l@mat.puc-rio.br
Subject: [obm-l] Provar que...
Date: Sat, 20 Dec 2014 04:44:26 +0300




100!  50^100, não estou conseguindo galera. Um abraço Carlos Gomes.
  
--

Esta mensagem foi verificada pelo sistema de antivírus e 

 acredita-se estar livre de perigo.   
-- 
Esta mensagem foi verificada pelo sistema de antivírus e
 acredita-se estar livre de perigo.



RE: [obm-l] Provar que...

2014-12-19 Por tôpico Eduardo Henrique
Menos  (50*51), esse é maior do que 50^2 
:)
Edu

From: dr.dhe...@outlook.com
To: obm-l@mat.puc-rio.br
Subject: RE: [obm-l] Provar que...
Date: Sat, 20 Dec 2014 05:14:46 +0300




Tenta reagrupar 100!, talvez algo como (1*100)(2*99)(3*98)...(50*51), dai você 
terá 50 produtos, cada um deles é equiparável a 50² (a saber menor), dai tem 
que argumentar um pouquinho, mas acho que sai.
AbraçosEdu

From: maikinho0...@hotmail.com
To: obm-l@mat.puc-rio.br
Subject: [obm-l] Provar que...
Date: Sat, 20 Dec 2014 04:44:26 +0300




100!  50^100, não estou conseguindo galera. Um abraço Carlos Gomes.
  
--

Esta mensagem foi verificada pelo sistema de antivírus e 

 acredita-se estar livre de perigo. 
  
-- 
Esta mensagem foi verificada pelo sistema de antivírus e
 acredita-se estar livre de perigo.



[obm-l] Exemplo de Cálculo

2014-12-18 Por tôpico Eduardo Henrique
Pessoas, estou aqui estudando o livro 2 do Guidorizzi de Cálculo e num problema 
ele fala que: 
Seja A é subconjunto de R², f:A-R² é tal que as derivadas parciais em relação 
a x e y são 0 para (x,y) em A. Dê um exemplo que a função f não seja constante.
Poderiam me ajudar?
Att.Eduardo   
-- 
Esta mensagem foi verificada pelo sistema de antivírus e
 acredita-se estar livre de perigo.



[obm-l] funções injetivas

2014-11-20 Por tôpico Eduardo Henrique
Olá pessoal, tudo bem?
Minha dúvida é de cunho teórico. Há, no âmbito de funções de duas variáveis (ou 
mais) f(x,y), alguma generalização do conceito de injetividade, sobrejetividade 
e bijetividade?
Att.Eduardo   
-- 
Esta mensagem foi verificada pelo sistema de antivírus e
 acredita-se estar livre de perigo.



[obm-l] Sequência

2014-11-19 Por tôpico Eduardo Henrique
Pessoas, estou com as seguintes dúvidas:1) Dada uma sequencia {Xn} de números 
reais, como eu posso criar uma nova sequencia partindo do 'k-ésimo' termo da 
sequencia anterior? (No caso, eu quero que a nova sequência tenha o primeiro 
termo igual ao k-ésimo termo duma sequencia anterior dada, e que o segundo 
termo da nova sequência seja igual ao (k+1)-ésimo termo e assim por diante).
2) Ache uma fórmula explicita para a sequência {1,0,0,1,0,0,1,0,0,1,0...}.
Att.Eduardo   
-- 
Esta mensagem foi verificada pelo sistema de antivírus e
 acredita-se estar livre de perigo.



[obm-l] Demonstração

2014-09-21 Por tôpico Eduardo Henrique
E ai pessoal, to aqui vendo um pouquinho de Análise e tem o seguinte teorema 
(cuja demonstração oficial está aqui e eu entendi) que eu dei uma demonstração. 
Gostaria que me dissessem se há algo de errado com a argumentação e se sim, 
dicas de como melhorar ela.
OBS1: Escrevi em linguagem de Latex pois é costume meu, qualquer dúvida quanto 
a linguagem só perguntar.

Teorema: Seja A \subset I_{n} ( I_{n} = {p \in N tal que 1 \leq p \leq n} ) tal 
que A \neq I_{n}. (Aqui uma observação, o número de elementos de I_{n} é n 
enquanto que o número de elementos de A e m, com m  n). Provar que não existe 
uma bijeção entre I_{n} e A.
Demonstração.
Seja f:I_{n} \rightarrow A uma função tal que f(1)=x_{1}, ... , f(m)=x_{m}, ... 
, f(n)=x_{n}.Se x_{i} \neq x_{j} quando i \neq j, então temos que o conjunto A 
tem, pelo menos, n elementos distintos, e como A \subset I_{n} e A \neq I_{n} 
temos uma primeira contradição.Se x_{i} = x_{j} quando i \neq j então temos que 
a função não é injetiva, logo não é bijetiva. Também contradição. Se x_{i} \neq 
x_{j} quando i = j, f não é função.
Logo, segue a tese.
OBS2: Dicas de como melhorar minha escrita são super bem vindas também.
Att.Eduardo   
-- 
Esta mensagem foi verificada pelo sistema de antivírus e
 acredita-se estar livre de perigo.



[obm-l] PBO

2014-09-14 Por tôpico Eduardo Henrique
Fiz uma demonstraçãozinha do PBO, queria opiniões por favor.
Teorema: Seja A \in N um conjunto não-vazio. A tem um menor elemento.
Demonstração:
Suponho que A seja um conjunto não-vazio de números naturais sem um menor 
elemento.Seja p_{0} \in A. Como A não possui menor elemento, temos que existe 
p_{1} menor que p_{0} em A. Raciocinando analogamente, temos uma cadeia 
infinita de números naturais menores do que p_{0} em A, mas isso é contradição 
pois o conjunto Nn={p \in N | 1 \leq p \leq p_{0}} é finito. Logo A é vazio
  
-- 
Esta mensagem foi verificada pelo sistema de antivírus e
 acredita-se estar livre de perigo.



[obm-l] RE: [obm-l] Uma fórmula

2014-06-09 Por tôpico Eduardo Henrique
a primeira creio que (n(n+1)/2)^10. A segunda sem ideias.

From: marconeborge...@hotmail.com
To: obm-l@mat.puc-rio.br
Subject: [obm-l] Uma fórmula
Date: Tue, 10 Jun 2014 01:40:25 +




Como calcular ( 1 + 2 + 3 + ... + 100)^10 e 1^10 + 2^10 + 3^10 + ... + 100^10 ? 
  
--

Esta mensagem foi verificada pelo sistema de antivírus e 

 acredita-se estar livre de perigo.   
-- 
Esta mensagem foi verificada pelo sistema de antivírus e
 acredita-se estar livre de perigo.



RE: [obm-l] Integrabilidade de Riemann

2014-06-04 Por tôpico Eduardo Henrique
Tendo em vista a ideia intuitiva de integral como area, nao ha de forma alguma 
contradicao. Ter ou nao primitiva (como composicao de funcoes elementares tipo 
polinomios, exponenciais, funcoes trigonometricas) eh um fator menor (e num 
geral voce nao consegue... A funcao distribuicao normal de Gauss eh um exemplo 
disso, pois ela soh se expressa com o simbolo da integral). Divida o intervalo 
em 2 e seja feliz. Integre em cada intervalo e some os resultados. PS: Desculpe 
a formatacao ruim do texto e a falta de acentos, meu celular nao permite algo 
muito melhor. Att. Eduardo

From: joao_maldona...@hotmail.com
To: obm-l@mat.puc-rio.br
Subject: [obm-l] Integrabilidade de Riemann
Date: Wed, 4 Jun 2014 21:54:14 -0300




Fala galera, vou ter prova de cálculo sexta e fiquei com uma dúvida de 
integrabilidade. Tem como vocês me darem uma ajuda?

Dada a função f:[0, 2]-R tal que f(x) = {1 se x1, 2 se x=1}
Determine se a função é integrável, e em caso positivo, ache sua  primitiva.

Tem um teorema que diz que, se f:[a,b] - R é contínua exceto num número finito 
de pontos, então f é integrável em [a, b]
Logo a função f é integrável, pois só é descontínua em x=1. 

Tem outro teorema que diz que, se f[a, b] - R tem descontinuidade tipo salto, 
isto é, existe c pertencente a (a, b) tal que o limite de f(x) quando x tende a 
c+ é diferente do limite de f(x) quando x tente a c-, então f não admite 
primitiva no intervalo [a, b].
E como no ponto x=1 tem descontinuidade tipo salto, então f(x) não admite 
primitiva em [0, 2]

f(x) seria integrável e não admitiria primitiva, absurdo!

Onde está o erro nessa demonstração? 

[]'s
João
  
--

Esta mensagem foi verificada pelo sistema de antivírus e 

 acredita-se estar livre de perigo.


  
-- 
Esta mensagem foi verificada pelo sistema de antivírus e
 acredita-se estar livre de perigo.



Re: [obm-l] Link caronet

2014-04-16 Por tôpico Eduardo Correa
Também gostaria de receber o link.
Obrigado


2014-04-15 21:47 GMT-03:00 Raphael Aureliano raphael0...@gmail.com:

 Se possível,  também gostaria do link.
 Muito obrigado.

 Raphael Aureliano

 Praticante de Oficial de Náutica (Piloto)
 Guarda-Marinha (RM-2)
 Em 15/04/2014 20:13, Jorge Paulino jorge...@yahoo.com.br escreveu:

  Também tenho interesse.

 Grato,

 Jorge


 Em 15/04/2014 18:06, Graciliano Antonio Damazo escreveu:

  Regis eu também gostaria.
 Abraços

Em Terça-feira, 15 de Abril de 2014 16:34, Vanderlei Nemitz
 vanderma...@gmail.com vanderma...@gmail.com escreveu:
   Eu também, obrigado!


 Em 15 de abril de 2014 15:48, Igor Battazza batta...@gmail.comescreveu:

 Boa tarde Regis,

  Gostaria do link.

  Abs,
 Igor


 Em 15 de abril de 2014 15:17, regis barros 
 regisgbar...@yahoo.com.brescreveu:

   Olá Pessoal
 Para aqueles que enviaram e-mail para mim já enviei o link com o
 caronnet, alguns dos livros estão em francês mas nada que o google tradutor
 não resolva e outros, ou seja, a grande maioria em português publicado na
 década de 50. Total de livros 9 volumes.
 Quando o pessoal da lista cita algum livro sempre vou dar uma olhada se
 tenho no meu acervo, ou compro o livro no estante virtual, caso seja muito
 raro de se encontrar pego na faculdade e ai faço um scan dele para ter na
 forma digital ou mesmo uma xerox resolve o problema, mas digo que os livros
 do caronnet são os mais dificil de se encontrar e realmente vale apena
 tê-los em casa.
 Algumas criticas é que o pessoal coloca os problemas ou cita eles e não
 diz da onde retirou o problema e assim fica díficil de ajudar ou mesmo que
 estamos conversando a mesma lingua.
 Hoje sei que na net tem um bilhão de problemas de olimpiadas de
 matematica e ai procurar eles e resolve-los é bem divertido, mas tem outras
 pessoas que não tem tempo para ficar na net garimpando os problemas e assim
 um site que eu conheço outra pessoa não conhece e assim podemos divulga-los
 para os demais colegas da lista.
 Não é a primeira vez que o livro do caronnet é citado na lista e alguns
 anos anteriores já vi esta citação e ai fui na captura dos livros logo é o
 resultado que alguns de vocês estão recebendo. Logo divulgem o link para os
 demais colegas.

  Uma abração

  RegisGBarros

 --
 Esta mensagem foi verificada pelo sistema de antivírus e
 acredita-se estar livre de perigo.



 --
 Esta mensagem foi verificada pelo sistema de antivírus e
 acredita-se estar livre de perigo.



 --
 Esta mensagem foi verificada pelo sistema de antiv�us e
 acredita-se estar livre de perigo.



 --
 Esta mensagem foi verificada pelo sistema de antiv�rus e
 acredita-se estar livre de perigo.



 --
 Esta mensagem foi verificada pelo sistema de antivírus e
 acredita-se estar livre de perigo.


 --
 Esta mensagem foi verificada pelo sistema de antivírus e
 acredita-se estar livre de perigo.




-- 
Professor Edu Corrêa

-- 
Esta mensagem foi verificada pelo sistema de antiv�rus e
 acredita-se estar livre de perigo.



Re: [obm-l] Questão difícil de combinatória - Campeonato Cearense de Futebol

2014-03-21 Por tôpico Luís Eduardo Háteras
Ola douglas,
Obrigado pela resolucao, mas nao estou concordando com a ultima 
expressao:[C(12,2).C(10,2).C(8,2).C(6,2).C(4,2).C(2,2)]/6! 
-[C(10,2).C(8,2).C(6,2).C(4,2).C(2,2)]/5! 
Pois quando vc escreve isso esta retirando os casos que fortaleza joga com 
ceara,mas o problema nao quer isso, eu interpretei que o problema esta 
interessado no ultimo jogo do primeiro turno que tem que ser fortaleza e ceara, 
entao, a ideia que vc usou (acredito) nao corresponde a realidade do problema...

Enviado via iPad

 Em 21/03/2014, às 10:51, douglas.olive...@grupoolimpo.com.br escreveu:
 
 Em 20.03.2014 23:38, Luís Eduardo Háteras escreveu:
 
 Sabendo-se que o campeonato cearense de futebol é disputado por 12 clubes, 
 entre os quais fortaleza e ceará.
 (A,B,C,D,E,F,G,H,I,J,FORTALEZA E CEARÁ)
 Determine a quantidade de maneiras de ocorrer o primeiro turno,
 PRIMEIRO TURNO 6 GRUPOS DE 2 EX: 
 (A,B);(C,D);(E,FORTALEZA);(F,CEARÁ);(G,H);(I,J) 
 COMO VEMOS FORTALEZA E CEARÁ NÃO ESTÃO JUNTOS
 de modo que o confronto entre fortaleza e ceara seja apenas na rodada final. 
 ENTENDI QUE ELES SÓ JOGARIAM JUNTOS NO FINAL E QUE SÓ INTERESSA A RESPOSTA 
 PARA O PRIMEIRO TURNO.
 Assim é só pegar 12 times e dividir em 6 grupos de 2 e retirar dessas 
 escolhas aquelas em que fortaleza e ceará estejam juntos.
 [C(12,2).C(10,2).C(8,2).C(6,2).C(4,2).C(2,2)]/6! 
 -[C(10,2).C(8,2).C(6,2).C(4,2).C(2,2)]/5! =Resposta
 
 -- 
 Esta mensagem foi verificada pelo sistema de antivírus e 
 acredita-se estar livre de perigo.
  
 
  
 
 -- 
 Esta mensagem foi verificada pelo sistema de antivírus e 
 acredita-se estar livre de perigo.

-- 
Esta mensagem foi verificada pelo sistema de antiv�rus e
 acredita-se estar livre de perigo.



[obm-l] Questão difícil de combinatória - Campeonato Cearense de Futebol

2014-03-20 Por tôpico Luís Eduardo Háteras
Sabendo-se que o campeonato cearense de futebol é disputado por 12 clubes, 
entre os quais fortaleza e ceará. Determine a quantidade de maneiras de ocorrer 
o primeiro turno, de modo que o confronto entre fortaleza e ceara seja apenas 
na rodada final. 
-- 
Esta mensagem foi verificada pelo sistema de antivírus e
 acredita-se estar livre de perigo.



[obm-l] Re: [obm-l] Re: [obm-l] Re: [obm-l] RE: [obm-l] Pentágono regular

2014-01-24 Por tôpico Eduardo Wilner
Pentágono ABCDE. 

Caso I. Todos ângulos congruentes sucessivos (seja A =  B  =  C)

AC=AB=DB (*) pois os triângulos ABC, BCD e ABE são congruentes.
Assim, são congruentes os ângulos (1)  BCA = BEA (= CDB (**));

Portanto, os triângulos BCE e ACE são congruentes, logo 
(2)  ACE = CEB .

O triângulo CDE é isoceles, implicando em (3) ECD = DEC .

Somando membro a membro (1)+(2)+(3) obtemos C = E.

De (*) o triângulo BDE é isoceles logo BDE = DEB que somado membro a membro 
com CDB = BEA de (*) nos dá D = E


Caso II. Ângulos congruentes não todos sucessivos (seja E = A = C).

Análogamente ao Caso I  EB=AD=BD e(***) EAD = DBC (por congruência dos 
triângulos isoceles que os tem como base). 
Triângulo isoceles ABD =  BAD = ABD e somando membro a membro com (***) =  
A = B. 

Triângulo isoceles EBD = EDB = BED, somado com BDC = BEA dá

D = E.


[ ]s

       





Em Quinta-feira, 23 de Janeiro de 2014 17:25, Eduardo Wilner 
eduardowil...@yahoo.com.br escreveu:
 
Não seria conveniente especificar que o pentágono é convexo (caso contrário a 
afirmação não seria válida)?

[ ]s





Em Quarta-feira, 22 de Janeiro de 2014 9:15, luiz silva 
luizfelipec...@yahoo.com.br escreveu:
 
Sugestão : coloque o pentágono dentro de uma circunferência, e veja o que 
acontece com os outros ângulos (quadriláteros inscritiveis).

Abs
Felipe





Em Terça-feira, 21 de Janeiro de 2014 23:05, marcone augusto araújo borges 
marconeborge...@hotmail.com escreveu:
 
Eu tentei mais algumas vezes e não consegui.Peço ajuda.
 



From: marconeborge...@hotmail.com
To: obm-l@mat.puc-rio.br
Subject: [obm-l] Pentágono regular
Date: Sat, 18 Jan 2014 18:58:08 +

 
Prove que um pentágono de lados congruentes e 3 ângulos congruentes é regular 
-- 
Esta mensagem foi verificada pelo sistema de antivírus e 
acredita-se estar livre de perigo.

-- 
Esta mensagem foi verificada pelo sistema de antivírus e 
acredita-se estar livre de perigo. 


-- 
Esta mensagem foi verificada pelo sistema de antivírus e 
acredita-se estar livre de perigo. 


-- 
Esta mensagem foi verificada pelo sistema de antivírus e 
acredita-se estar livre de perigo. 
-- 
Esta mensagem foi verificada pelo sistema de antivírus e
 acredita-se estar livre de perigo.



[obm-l] Re: [obm-l] Re: [obm-l] RE: [obm-l] Pentágono regular

2014-01-23 Por tôpico Eduardo Wilner
Não seria conveniente especificar que o pentágono é convexo (caso contrário a 
afirmação não seria válida)?

[ ]s





Em Quarta-feira, 22 de Janeiro de 2014 9:15, luiz silva 
luizfelipec...@yahoo.com.br escreveu:
 
Sugestão : coloque o pentágono dentro de uma circunferência, e veja o que 
acontece com os outros ângulos (quadriláteros inscritiveis).

Abs
Felipe





Em Terça-feira, 21 de Janeiro de 2014 23:05, marcone augusto araújo borges 
marconeborge...@hotmail.com escreveu:
 
Eu tentei mais algumas vezes e não consegui.Peço ajuda.
 



From: marconeborge...@hotmail.com
To: obm-l@mat.puc-rio.br
Subject: [obm-l] Pentágono regular
Date: Sat, 18 Jan 2014 18:58:08 +

 
Prove que um pentágono de lados congruentes e 3 ângulos congruentes é regular 
-- 
Esta mensagem foi verificada pelo sistema de antivírus e 
acredita-se estar livre de perigo.

-- 
Esta mensagem foi verificada pelo sistema de antivírus e 
acredita-se estar livre de perigo. 


-- 
Esta mensagem foi verificada pelo sistema de antivírus e 
acredita-se estar livre de perigo. 
-- 
Esta mensagem foi verificada pelo sistema de antivírus e
 acredita-se estar livre de perigo.



Re: [obm-l] Como eu resolvo isso?

2014-01-06 Por tôpico Eduardo Wilner
Maldonado, poderia nos mostrar o problema original, de cinemática?

[ ]'s  





Em Sábado, 4 de Janeiro de 2014 18:08, saulo nilson saulo.nil...@gmail.com 
escreveu:
 
essa integral e catalogada caiu em uma prova da obmu.



2014/1/4 saulo nilson saulo.nil...@gmail.com

d2R/R=2d2acosa/sena
lnR dR=2(lnsena+V/D)da
RlnR-R+D=2aV/D+2Integral (lnsenada)



2014/1/3 João Maldonado joao_maldona...@hotmail.com

Fala ai galera. Eu tava resolvendo um problema de cinemática (sei que não é o 
assunto da lista) mas caí numa parte puramente matemática que não estou 
conseguindo resolver, queria pedir a ajuda de vocês. Se alguém puder me dar 
uma mão eu agradeço muito


d²R/dt² = 2cos(a)/R³
R d²a/dt² = sen(a)/R³


onde: R inicial = D
a inicial = 0
dR/dt inicial = 0
da/dt inicial = V/D


Quero achar R(t), a(t) e R(a)


[]'s
João
-- 
Esta mensagem foi verificada pelo sistema de antivírus e 
acredita-se estar livre de perigo. 



-- 
Esta mensagem foi verificada pelo sistema de antivírus e 
acredita-se estar livre de perigo. 
-- 
Esta mensagem foi verificada pelo sistema de antivírus e
 acredita-se estar livre de perigo.



Re: [obm-l] Coordenadas polares

2013-11-30 Por tôpico Eduardo Wilner
A secante é negativa no segundo e terceiro quadrante e rhô não pode ser 
negativo.

[ ]'s 




Em Sábado, 30 de Novembro de 2013 0:06, marcone augusto araújo borges 
marconeborge...@hotmail.com escreveu:
 
Eu nao entendi bem essa variação de teta.




Date: Sun, 24 Nov 2013 06:56:28 -0800
From: eduardowil...@yahoo.com.br
Subject: Re: [obm-l] Coordenadas polares
To: obm-l@mat.puc-rio.br


Como rô=0 , -pi/2= teta = pi/2 (à menos de k2pi);

Assim rô*cos(teta) = 1, que é a projeção de rô no eixo polar... tá enxergando?


[ ]'s




Em Domingo, 24 de Novembro de 2013 10:26, Bernardo Freitas Paulo da Costa 
bernardo...@gmail.com escreveu:
 
2013/11/24 marcone augusto araújo borges marconeborge...@hotmail.com

 Alguém poderia mostrar como fica o gráfico de rô = sec(teta)?

Transforme r e teta em coordenadas cartesianas, r^2 = x^2 + y^2,
tan(teta) = y/x, e depois substitua um pouco de trigonometria.

Abraços,
-- 
Bernardo Freitas Paulo da Costa


-- 
Esta mensagem foi verificada pelo sistema de antivírus e
acredita-se estar livre de perigo.


=
Instruções para entrar na lista, sair da lista e usar a lista em
http://www.mat.puc-rio.br/~obmlistas/obm-l.html

=



-- 
Esta mensagem foi verificada pelo sistema de antivírus e 
acredita-se estar livre de perigo.
-- 
Esta mensagem foi verificada pelo sistema de antivírus e 
acredita-se estar livre de perigo. 
-- 
Esta mensagem foi verificada pelo sistema de antivírus e
 acredita-se estar livre de perigo.



Re: [obm-l] Coordenadas polares

2013-11-24 Por tôpico Eduardo Wilner
Como rô=0 , -pi/2= teta = pi/2 (à menos de k2pi);

Assim rô*cos(teta) = 1, que é a projeção de rô no eixo polar... tá enxergando?


[ ]'s




Em Domingo, 24 de Novembro de 2013 10:26, Bernardo Freitas Paulo da Costa 
bernardo...@gmail.com escreveu:
 
2013/11/24 marcone augusto araújo borges marconeborge...@hotmail.com

 Alguém poderia mostrar como fica o gráfico de rô = sec(teta)?

Transforme r e teta em coordenadas cartesianas, r^2 = x^2 + y^2,
tan(teta) = y/x, e depois substitua um pouco de trigonometria.

Abraços,
-- 
Bernardo Freitas Paulo da Costa


-- 
Esta mensagem foi verificada pelo sistema de antivírus e
acredita-se estar livre de perigo.


=
Instruções para entrar na lista, sair da lista e usar a lista em
http://www.mat.puc-rio.br/~obmlistas/obm-l.html

=
-- 
Esta mensagem foi verificada pelo sistema de antivírus e
 acredita-se estar livre de perigo.



Re: [obm-l] Coordenadas polares

2013-11-24 Por tôpico Eduardo Wilner
Ninguém achou a minha proposta mais simples? 





Em Domingo, 24 de Novembro de 2013 22:26, Bernardo Freitas Paulo da Costa 
bernardo...@gmail.com escreveu:
 
2013/11/24 marcone augusto araújo borges marconeborge...@hotmail.com:
 r = sec(teta)
 r^2 = [sec(teta)]^2 = 1 + [tg(teta)]^2
 x^2 + y^2 = 1 + (y/x)^2
 E dai?

Daí que

x^2 + y^2 = 1 + (y/x)^2 = (x^2 + y^2)/x^2, logo x^2 = 1 (pois r != 0,
já que a secante é sempre diferente de zero, logo podemos cortar x^2 +
y^2). Daí, x = 1 ou x = -1, e mais um pouco de trigonometria vai
mostrar que x = -1 é uma solução espúria, introduzida porque a gente
elevou ao quadrado (isso quase sempre acontece, e é sempre bom
verificar como... aqui, se fosse r = - sec(teta), teria dado a mesma
depois de elevar ao quadrado, portanto claro que uma das soluções tem
que ser desprezada). Isso quer dizer que x = 1, ou seja, e é equação
de uma reta vertical.


Abraços,
-- 
Bernardo Freitas Paulo da Costa

-- 
Esta mensagem foi verificada pelo sistema de antivírus e
acredita-se estar livre de perigo.


=
Instruções para entrar na lista, sair da lista e usar a lista em
http://www.mat.puc-rio.br/~obmlistas/obm-l.html
=
-- 
Esta mensagem foi verificada pelo sistema de antivírus e
 acredita-se estar livre de perigo.



[obm-l] Re: [obm-l] Cônicas

2013-11-24 Por tôpico Eduardo Wilner
Uma translação dos eixos de forma que a origem coincida com o ponto dado 
(10;25) deixa o trabalho de encontrar m(1;-29/4) mais agradável.



[ ]'s




Em Domingo, 24 de Novembro de 2013 8:26, marcone augusto araújo borges 
marconeborge...@hotmail.com escreveu:
 
 
Determinar as equações das retas tangentes à cônica x^2 + 4y^2 - 180 = 0
 
que passam pelo ponto (10,25)

Eu escrevi  y - 25 =m(x - 10) *
A ideia era substituir o valor de y em * na equação lá de cima e igualar delta 
a zero
para achar m,mas as contas ficaram enormes          
-- 
Esta mensagem foi verificada pelo sistema de antivírus e 
acredita-se estar livre de perigo. 
-- 
Esta mensagem foi verificada pelo sistema de antivírus e
 acredita-se estar livre de perigo.



[obm-l] Re: [obm-l] Re: [obm-l] Re: [obm-l] Re: [obm-l] Diferença de cubos

2013-11-11 Por tôpico Eduardo Wilner
Quando eu coloquei a errata achei que estivesse claro que eu já tinha 
entendido... Afinal, para bom entendedor meia palavra besta, ops, basta...

O que confundiu foi que como estava o segundo membro da igualdade era maior 
que o primeiro!

E a soma dos quadrados?


[]'s




Em Segunda-feira, 11 de Novembro de 2013 23:51, terence thirteen 
peterdirich...@gmail.com escreveu:
 
Isso mesmo, é só abrir:

(b+1)^3 - b^3 =3a^2+3a+1

3a^2+3a+1=b^2


3*(4a^2+4a+1)+1=4b^2
3(2a+1)^2+1=(2b)^2

Eu usei uma técnica de completar quadrados neste parêntese.

Eu multipliquei por 4 para deixar tudo par, mas se quiser, eis a forma fácil de 
entender:

3a^2+3a+1=b^2


a^2+a+1/3=b^2/3

a^2+2*1/2*a+(1/2)^2 - (1/2)^2 + 1/3=b^2/3


(a+1/2)^2 - 1/4 + 1/3=b^2/3


Tira o mínimo e voilà!















Em 11 de novembro de 2013 00:22, Eduardo Wilner eduardowil...@yahoo.com.br 
escreveu:

No segundo membro, onde se lê b, leia-se a ?






Em Segunda-feira, 11 de Novembro de 2013 0:07, Eduardo Wilner 
eduardowil...@yahoo.com.br escreveu:
 
Como se chega à   (2b)^2=3(2b+1)^2+1 ?


A propósito, a expressão parece estar incorreta.






Em Domingo, 10 de Novembro de 2013 19:01, terence thirteen 
peterdirich...@gmail.com escreveu:
 
Isto equivale a uma equação de Pell!


(a+1)^3-a^3=b^2 acarreta
(2b)^2=3(2b+1)^2+1


Talvez usando reciprocidade, fique mais fácil...



Em 9 de novembro de 2013 23:20, marcone augusto araújo borges 
marconeborge...@hotmail.com escreveu:

Mostre que se a diferença de dois cubos consecutivos é um quadrado,então   
  
é o quadrado da soma de dois quadrados.
8^3 - 7^3 = (3^2 + 2^2)^2
-- 
Esta mensagem foi verificada pelo sistema de antivírus e 
acredita-se estar livre de perigo. 



-- 
/**/
神が祝福

Torres 
-- 
Esta mensagem foi verificada pelo sistema de antiv�us e 
acredita-se estar livre de perigo. 





-- 
Esta mensagem foi verificada pelo sistema de antivírus e 
acredita-se estar livre de perigo. 


-- 
/**/
神が祝福

Torres 
-- 
Esta mensagem foi verificada pelo sistema de antiv�us e 
acredita-se estar livre de perigo. 
-- 
Esta mensagem foi verificada pelo sistema de antiv�rus e
 acredita-se estar livre de perigo.



[obm-l] Re: [obm-l] Re: [obm-l] Re: [obm-l] Re: [obm-l] Diferença de cubos

2013-11-11 Por tôpico Eduardo Wilner
O que confundiu foi que, como estava, o segundo membro da igualdade era maior 
que o primeiro!




Em Terça-feira, 12 de Novembro de 2013 1:15, Eduardo Wilner 
eduardowil...@yahoo.com.br escreveu:
 
Quando eu coloquei a errata achei que estivesse claro que eu já tinha 
entendido... Afinal, para bom entendedor meia palavra besta, ops, basta...

O que confundiu foi que como estava o segundo membro da igualdade era maior 
que o primeiro!

E a soma dos quadrados?


[]'s




Em Segunda-feira, 11 de Novembro de 2013 23:51, terence thirteen 
peterdirich...@gmail.com escreveu:
 
Isso mesmo, é só abrir:

(b+1)^3 - b^3 =3a^2+3a+1

3a^2+3a+1=b^2


3*(4a^2+4a+1)+1=4b^2
3(2a+1)^2+1=(2b)^2

Eu usei uma técnica de completar quadrados neste parêntese.

Eu multipliquei por 4 para deixar tudo par, mas se quiser, eis a forma fácil de 
entender:

3a^2+3a+1=b^2


a^2+a+1/3=b^2/3

a^2+2*1/2*a+(1/2)^2 - (1/2)^2 + 1/3=b^2/3


(a+1/2)^2 - 1/4 + 1/3=b^2/3


Tira o mínimo e voilà!















Em 11 de novembro de 2013 00:22, Eduardo Wilner eduardowil...@yahoo.com.br 
escreveu:

No segundo membro, onde se lê b, leia-se a ?






Em Segunda-feira, 11 de Novembro de 2013 0:07, Eduardo Wilner 
eduardowil...@yahoo.com.br escreveu:
 
Como se chega à   (2b)^2=3(2b+1)^2+1 ?


A propósito, a expressão parece estar incorreta.






Em Domingo, 10 de Novembro de 2013 19:01, terence thirteen 
peterdirich...@gmail.com escreveu:
 
Isto equivale a uma equação de Pell!


(a+1)^3-a^3=b^2 acarreta
(2b)^2=3(2b+1)^2+1


Talvez usando reciprocidade, fique mais fácil...



Em 9 de novembro de 2013 23:20, marcone augusto araújo borges 
marconeborge...@hotmail.com escreveu:

Mostre que se a diferença de dois cubos consecutivos é um quadrado,então   
  
é o quadrado da soma de dois quadrados.
8^3 - 7^3 = (3^2 + 2^2)^2
-- 
Esta mensagem foi verificada pelo sistema de antivírus e 
acredita-se estar livre de perigo. 



-- 
/**/
神が祝福

Torres 
-- 
Esta mensagem foi verificada pelo sistema de antiv�us e 
acredita-se estar livre de perigo. 





-- 
Esta mensagem foi verificada pelo sistema de antivírus e 
acredita-se estar livre de perigo. 


-- 
/**/
神が祝福

Torres 
-- 
Esta mensagem foi verificada pelo sistema de antiv�us e 
acredita-se estar livre de perigo. 
-- 
Esta mensagem foi verificada pelo sistema de antiv�rus e
 acredita-se estar livre de perigo.



[obm-l] Re: [obm-l] Re: [obm-l] Diferença de cubos

2013-11-10 Por tôpico Eduardo Wilner
Como se chega à   (2b)^2=3(2b+1)^2+1 ?

A propósito, a expressão parece estar incorreta.





Em Domingo, 10 de Novembro de 2013 19:01, terence thirteen 
peterdirich...@gmail.com escreveu:
 
Isto equivale a uma equação de Pell!

(a+1)^3-a^3=b^2 acarreta
(2b)^2=3(2b+1)^2+1

Talvez usando reciprocidade, fique mais fácil...



Em 9 de novembro de 2013 23:20, marcone augusto araújo borges 
marconeborge...@hotmail.com escreveu:

Mostre que se a diferença de dois cubos consecutivos é um quadrado,então
 
é o quadrado da soma de dois quadrados.
8^3 - 7^3 = (3^2 + 2^2)^2
-- 
Esta mensagem foi verificada pelo sistema de antivírus e 
acredita-se estar livre de perigo. 


-- 
/**/
神が祝福

Torres 
-- 
Esta mensagem foi verificada pelo sistema de antiv�us e 
acredita-se estar livre de perigo. 
-- 
Esta mensagem foi verificada pelo sistema de antiv�rus e
 acredita-se estar livre de perigo.



[obm-l] Re: [obm-l] Re: [obm-l] Diferença de cubos

2013-11-10 Por tôpico Eduardo Wilner
No segundo membro, onde se lê b, leia-se a ?





Em Segunda-feira, 11 de Novembro de 2013 0:07, Eduardo Wilner 
eduardowil...@yahoo.com.br escreveu:
 
Como se chega à   (2b)^2=3(2b+1)^2+1 ?

A propósito, a expressão parece estar incorreta.





Em Domingo, 10 de Novembro de 2013 19:01, terence thirteen 
peterdirich...@gmail.com escreveu:
 
Isto equivale a uma equação de Pell!

(a+1)^3-a^3=b^2 acarreta
(2b)^2=3(2b+1)^2+1

Talvez usando reciprocidade, fique mais fácil...



Em 9 de novembro de 2013 23:20, marcone augusto araújo borges 
marconeborge...@hotmail.com escreveu:

Mostre que se a diferença de dois cubos consecutivos é um quadrado,então
 
é o quadrado da soma de dois quadrados.
8^3 - 7^3 = (3^2 + 2^2)^2
-- 
Esta mensagem foi verificada pelo sistema de antivírus e 
acredita-se estar livre de perigo. 


-- 
/**/
神が祝福

Torres 
-- 
Esta mensagem foi verificada pelo sistema de antiv�us e 
acredita-se estar livre de perigo. 
-- 
Esta mensagem foi verificada pelo sistema de antiv�rus e
 acredita-se estar livre de perigo.



Re: [obm-l] comprimento de corda elicoidal

2013-11-07 Por tôpico Eduardo Wilner
O comprimento depende do passo da hélice que não foi especificado.
Para liberá-lo podemos acrescentar um fator q ao argumento das funções 
trigonométricas; assim, para q=2pi, p.ex., a hélice daria uma volta completa,

[ ]' s




Em Quinta-feira, 7 de Novembro de 2013 1:04, Eduardo Wilner 
eduardowil...@yahoo.com.br escreveu:
 
Parece ser uma hélice conica que pode ser expressa paramétricamente como

f(t)=[ t.cos t, t.sen t, e.(t-1)]  , supondo que o 2,7 seja o neperiano e.

 Daí é só integrar |f'(t)|.dt entre t=1 e t=2.


[ ]'s  




Em Quarta-feira, 6 de Novembro de 2013 11:01, claudinei claudin...@gmail.com 
escreveu:
 

Alguem por favor poderia me ajudar respondendo como calculo o comprimento
 de uma corda elicoidal de raio variável? Exemplo: uma corda elicoidal de raio 
da base 1 e raio superior 2 de altura 2,7?
 
Desde ja agradeço


-- 
Esta mensagem foi verificada pelo sistema de antivírus e 
acredita-se estar livre de perigo. 


-- 
Esta mensagem foi verificada pelo sistema de antivírus e 
acredita-se estar livre de perigo. 
-- 
Esta mensagem foi verificada pelo sistema de antivírus e
 acredita-se estar livre de perigo.



Re: [obm-l] comprimento de corda elicoidal

2013-11-06 Por tôpico Eduardo Wilner
Parece ser uma hélice conica que pode ser expressa paramétricamente como

f(t)=[ t.cos t, t.sen t, e.(t-1)]  , supondo que o 2,7 seja o neperiano e.

 Daí é só integrar |f'(t)|.dt entre t=1 e t=2.


[ ]'s  




Em Quarta-feira, 6 de Novembro de 2013 11:01, claudinei claudin...@gmail.com 
escreveu:
 

Alguem por favor poderia me ajudar respondendo como calculo o comprimento
 de uma corda elicoidal de raio variável? Exemplo: uma corda elicoidal de raio 
da base 1 e raio superior 2 de altura 2,7?
 
Desde ja agradeço


-- 
Esta mensagem foi verificada pelo sistema de antivírus e 
acredita-se estar livre de perigo. 
-- 
Esta mensagem foi verificada pelo sistema de antivírus e
 acredita-se estar livre de perigo.



[obm-l] O regresso da Elipse e círculo

2013-10-10 Por tôpico Eduardo Wilner
Há alguns dias Douglas Oliveira manifestou sua surpresa quando não conseguiu 
resolver o problema de tangencia usando geometria analítica e impondo 
discriminante nulo na equação da intersecção, e na ocasião alertei que o motivo 
era ter sido escolhido o foco para centro da circunferencia.

Na verdade, o foco não é o unico ponto que gera este problema.

Me parece interessante propor a questão: que pontos do (eixo maior da elipse) 
servem como centro, para que o método tentado por Douglas funcione? 



[ ]'s    

-- 
Esta mensagem foi verificada pelo sistema de antivírus e
 acredita-se estar livre de perigo.



Re: [obm-l] Quadrado perfeito

2013-10-08 Por tôpico Eduardo Wilner


(3x-4)^2 - 4 = n^2   , se m = 3x -4   = m^2 - n^2 = 4   ou  (m/2)^2  - (n/2)^2 
=1

Equação de Pell com parâmetro , 1, quadrado perfeito .   


Assim n=0  e m/2 = + ou - 1 = 3x -4 = + ou - 2   = x = 2 (ou 2/3 que não é 
inteiro).

[ ]'s
De: marcone augusto araújo borges marconeborge...@hotmail.com
Para: obm-l@mat.puc-rio.br obm-l@mat.puc-rio.br 
Enviadas: Terça-feira, 8 de Outubro de 2013 9:15
Assunto: [obm-l] Quadrado perfeito
 


 
Determine todos os valores inteiros positivos de x tais que 9x^2 - 24x + 12 

é um quadrado perfeito.
-- 
Esta mensagem foi verificada pelo sistema de antivírus e 
acredita-se estar livre de perigo. 
-- 
Esta mensagem foi verificada pelo sistema de antivírus e
 acredita-se estar livre de perigo.



[obm-l] Re: [obm-l] Elipse e círculo

2013-10-02 Por tôpico Eduardo Wilner
É um caso particular pelo fato da circunferência estar centrada no foco.

Observe que o ponto de tangência utilizando o raio encontrado com  geometria 
plana é o vértice 
da elipse, e lembre-se que a normal à elipse é bissetriz do ângulo com vértice 
no ponto da elise e com lados passando pelos focos.

[ ]'s  



 De: douglas.olive...@grupoolimpo.com.br douglas.olive...@grupoolimpo.com.br
Para: obm-l@mat.puc-rio.br 
Enviadas: Quarta-feira, 2 de Outubro de 2013 13:57
Assunto: [obm-l] Elipse e círculo
 


Caros amigos elaborei um exercício em sala de aula, com a intenção de resolver 
mais um problema de tangencia,
peguei uma elipse com eixo maior igual a 10cm e eixo menor igual a 6cm e nesta 
elipse inscrevi uma circunferência
com centro em um dos focos e fui calcular o raio desta circunferência para que 
ela fosse tangente á elipse
(por dentro) escrevi a equação da elipse e a equação da circunferência e usei a 
condição padrão de
tangencia( delta=zero) , mas para minha surpresa não deu muito certo e o raio 
fico igual a zero,
porém em outra solução (por geometria plana)
encontrei o raio igual a 1cm , e agora? Seria um caso particular? Alguém já 
passou por isso?
 
Att: Douglas Oliveira
 
 
 
-- 
Esta mensagem foi verificada pelo sistema de antiv�us e 
acredita-se estar livre de perigo. 
-- 
Esta mensagem foi verificada pelo sistema de antiv�rus e
 acredita-se estar livre de perigo.



[obm-l] Re: [obm-l] RE: [obm-l] Polinômios

2013-09-26 Por tôpico Eduardo Wilner
Oi Maldonado.

Gostaria de entender a notação:  

parece que cp seriam as raizes, mas, em cp=1/ap, ap seriam os coeficientes? 
Como?

[ ]'s





 De: João Maldonado joao_maldona...@hotmail.com
Para: obm-l@mat.puc-rio.br obm-l@mat.puc-rio.br 
Enviadas: Terça-feira, 24 de Setembro de 2013 23:00
Assunto: [obm-l] RE: [obm-l] Polinômios
 


 
Sendo cp = 1/ap
a1a2...an =  +-1/an
a1a2...an(1/a1 + 1/a2+...+1/an) =   -+1/an
a1a2...an(1/a1a2 + 1/a1a3 +... +1/an-1an) =   +-1/an

Logo: (1/a1 + 1/a2+...+1/an) = -1
(1/a1a2 + 1/a1a3 +... +1/an-1an) =1
x=c1+c2+ ... +cn = -1
y=c1c2+c1c3+...+cn-1cn = 1

c1²+c2² +... +cn² = (c1+c2+ ... +cn)² -2(c1c2+c1c3+...+cn-1cn) = -1, absurdo, 
logo para n par temos  que pelo menos 2 raízes são complexas

[]'s
João





From: marconeborge...@hotmail.com
To: obm-l@mat.puc-rio.br
Subject: [obm-l] Polinômios 
Date: Wed, 25 Sep 2013 01:00:58 +

 
Prove que um polinômio do tipo a(n)x^n + a(n-1)x^(n-1) + ...+ a(3)x^3 + x^2 + x 
+ 1 com 
coeficientes reais não pode ter todas as raízes reais.

-- 
Esta mensagem foi verificada pelo sistema de antivírus e 
acredita-se estar livre de perigo.
-- 
Esta mensagem foi verificada pelo sistema de antivírus e 
acredita-se estar livre de perigo. 
-- 
Esta mensagem foi verificada pelo sistema de antiv�rus e
 acredita-se estar livre de perigo.



Re: [obm-l] Como que faz??

2013-09-25 Por tôpico Eduardo Wilner
Oi Carlos.

No item 2) vc. diz que CGE = x+y; isto significaria,  CGE = ADE . Vc. 
poderia explicar?

Obrigado

[ ]'s





 De: Carlos Victor victorcar...@globo.com
Para: obm-l@mat.puc-rio.br 
Enviadas: Terça-feira, 24 de Setembro de 2013 19:30
Assunto: Re: [obm-l] Como que faz??
 


Olá Douglas,
Acredito ter conseguido uma resolução para o problema 2 de geometria que vc 
postou aqui  .

Vamos lá  e acompanhe fazendo a figura , ok ?
vamos provar que na verdade o ângulo DEF é o dobro de ADC.
Seja  o ângulo  ADC = x e o ângulo CDE = y .

1) Trace CE e observe que o quadrilátero ACED é inscritível . então AEC = x  e  
EAC = y .

2) seja G a intersecção de CD com a circunferência C´ . Trace  EG e observe que 
 o ângulo CGE = x + y . Daí concluímos que o ângulo GED = x .

3) Não é difícil de mostrar que EB  é bissetriz de AEG . Seja então os ângulos 
AEB= DEB = z .

4) Trace agora a perpendicular de B  ao segmento ED e seja H o pé desta 
perpendicular. Observe que o quadrilátero BFHD é inscritível , então BHF = x 
.Trace FH e observe que EG é perpendicular  a FH . Seja J a intersecção de FH 
com EG .

5) Como o triângulo CEG está inscrito na circunferência C´ e observando que BF 
é perpendicular ao lado CD , pelo enunciado ; teremos pela reta de SIMSON , que 
 os pés das  perpendiculares traçadas de B aos lados CG , EG e EC  estão 
alinhados. Sejam então I o pé da perpendicular traçada de ao lado EG  e R o pé  
da perpendicular traçada de B ao lado CE .

6) observando os quadriláteros inscritíveis : BIER , BIHE , teremos q










Em 23 de agosto de 2013 16:03, douglas.olive...@grupoolimpo.com.br escreveu:

 
Olá , alguns alunos do ensino médio da instituição onde trabalho me deram 
alguns problemas do site https://brilliant.org/
E não consegui achar solução para dois deles, vou escreve-los abaixo e se 
alguém puder me ajudar agradeço.
 
PROBLEMA 1: Dada uma função f:R-R tal que f(2x^2 -1)=2(f(x))^2 -1 e f(x) é um 
polinômio de grau 13, sendo assim determine o coeficiente de x^5 de f(x).
 
PROBLEMA 2: Seja uma circunferência C'  e um ponto externo A , traça-se por A 
duas tangentes a circunferência que a interceptam nos pontos B e C , marca-se 
no prolongamento de AB no sentido de A para B um ponto D tal que o ângulo 
ADC=25 graus, traça-se por B uma perpendicular ao segmento CD que intercepta 
CD em F . Agora considere um outra circunferência C'' circunscrita ao 
triângulo ADC que intercepta a primeira circunferência C' no ponto E . 
Determinar a medida do ângulo DEF.
 
 
Obs: Fiz a segunda figura no geogebra e encontrei 50 graus como resposta , 
preciso na verdade de uma resolução. 
 
Att, Douglas Oliveira. 
-- 
Esta mensagem foi verificada pelo sistema de antivírus e 
acredita-se estar livre de perigo. 

-- 
Esta mensagem foi verificada pelo sistema de antivírus e 
acredita-se estar livre de perigo. 
-- 
Esta mensagem foi verificada pelo sistema de antivírus e
 acredita-se estar livre de perigo.



[obm-l] Re: [obm-l] Re: [obm-l] Re: [obm-l] Álgebra(não tá saindo)

2013-09-18 Por tôpico Eduardo Wilner
Não seria n = 5 + 6k ? Apenas para fins de clareza e precisão, pois no caso 
interessa apenas o mínimo (5)...

[ ]'s 





 De: Willy George Amaral Petrenko wgapetre...@gmail.com
Para: obm-l@mat.puc-rio.br 
Enviadas: Sábado, 14 de Setembro de 2013 23:34
Assunto: [obm-l] Re: [obm-l] Re: [obm-l] Álgebra(não tá saindo)
 


Ou resolva a equação em N:

(10*x+6)*4 = 6*10n + x ⇒ 39*x + 24 = 6*10n ⇒ 13*x = 2*10n - 8 ⇒ 10n = 4 mod 13 
⇒ n = 5 + 12k. Logo o menor n é 5 e o menor número é (2*105 - 8)/13 = 15384  
Obviamente vc adiciona o 6 depois: 153846



2013/9/14 Ralph Teixeira ralp...@gmail.com

Escreva a multiplicacao que nem a gente fazia lah na 4a serie:
_6
              x4
6_


Agora vah fazendo a multiplicacao. 6x4=24, entao poe o 4, vai 2.
Mas, se eh 4 ali embaixo, eh 4 do lado esquero do 6. Entao fica algo assim:
46
               x4
64Agora 4x4=16, mais 2, dah 18. Entao poe o 8 no resultado E TAMBEM DO 
LADO DO 4 NA PRIMEIRA LINHA (e vai 1).
___846

               x4
6___84
4x8=32, +1=33. O proximo eh 3. Continue assim achando os digitos da direita 
para a esquerda: 5, 1... E entao o proximo eh 6, que PODE ser aquele 6 
inicial! 


Assim, o menor numero inteiro n eh 153846.


Abraco,
        Ralph





2013/9/14 marcone augusto araújo borges marconeborge...@hotmail.com

Encontre o menor inteiro positivo n que possui as seguintes propriedades:  
   
I. Em sua representação tem o 6 como último dígito
II.Se o último dígito(6) é apagado  e colocado na frente dos dígitos 
restantes,o número resultante
é quatro vezes maior que o número original n
-- 
Esta mensagem foi verificada pelo sistema de antivírus e 
acredita-se estar livre de perigo. 

-- 
Esta mensagem foi verificada pelo sistema de antivírus e 
acredita-se estar livre de perigo. 

-- 
Esta mensagem foi verificada pelo sistema de antivírus e 
acredita-se estar livre de perigo. 
-- 
Esta mensagem foi verificada pelo sistema de antiv�rus e
 acredita-se estar livre de perigo.



[obm-l] Re: [obm-l] Re: [obm-l] Re: [obm-l] Re: [obm-l] Álgebra(não tá saindo)

2013-09-16 Por tôpico Eduardo Wilner
x tem que ser par: seja x=2y = 10n = 13*y + 4 ...

[ ]'s





 De: Hermann ilhadepaqu...@bol.com.br
Para: obm-l@mat.puc-rio.br 
Enviadas: Domingo, 15 de Setembro de 2013 11:18
Assunto: [obm-l] Re: [obm-l] Re: [obm-l] Re: [obm-l] Álgebra(não tá saindo)
 


 
Poderiam me explicar essa passagem
 13*x = 2*10n - 8 ⇒ 10n = 4 mod 13 
obrigado
 Hermann
- Original Message - 
From: Willy  George Amaral Petrenko 
To: obm-l@mat.puc-rio.br 
Sent: Saturday, September 14, 2013 11:34  PM
Subject: [obm-l] Re: [obm-l] Re: [obm-l]  Álgebra(não tá saindo)


Ou resolva a equação em N: 


(10*x+6)*4 = 6*10n + x ⇒ 39*x + 24 = 6*10n ⇒ 13*x =  2*10n - 8 ⇒ 10n = 4 mod 
13 ⇒ n = 5 + 12k. Logo o menor n  é 5 e o menor número é (2*105 - 8)/13 = 
15384   Obviamente vc adiciona o 6 depois: 153846



2013/9/14 Ralph Teixeira ralp...@gmail.com

Escreva a multiplicacao que nem a gente fazia lah na 4a serie:
_6
              x4
6_


Agora vah fazendo a multiplicacao. 6x4=24, entao poe o 4, vai 2.
Mas, se eh 4 ali embaixo, eh 4 do lado esquero do 6. Entao fica algo  assim:
46
               x4
64Agora 4x4=16, mais 2, dah 18. Entao poe o 8 no  resultado E TAMBEM 
DO LADO DO 4 NA PRIMEIRA LINHA (e vai 1). 
___846

               x4
6___84
4x8=32, +1=33. O proximo eh 3. Continue assim achando os digitos da  direita 
para a esquerda: 5, 1... E entao o proximo eh 6, que PODE ser aquele  6 
inicial!  


Assim, o menor numero inteiro n eh 153846.


Abraco,
        Ralph





2013/9/14 marcone augusto araújo borges marconeborge...@hotmail.com

Encontre o menor inteiro positivo n que possui as seguintes  propriedades:
  
I. Em sua representação tem o 6 como último dígito
II.Se o último dígito(6) é apagado  e colocado na frente dos  dígitos 
restantes,o número resultante
é quatro vezes maior que o número original n
-- 
Esta mensagem foi verificada pelo sistema de antivírus e 
acredita-se estar livre de perigo. 

-- 
Esta mensagem 
foi verificada pelo sistema de antivírus e 
acredita-se estar livre de 
perigo. 

-- 
Esta mensagem foi 
  verificada pelo sistema de antivírus e 
acredita-se estar livre de perigo. 
-- 
Esta mensagem foi verificada pelo sistema de antivírus e 
acredita-se estar livre de perigo. 
-- 
Esta mensagem foi verificada pelo sistema de antiv�rus e
 acredita-se estar livre de perigo.



[obm-l] Re: [obm-l] RE: [obm-l] RE: [obm-l] RE: [obm-l] RE: [obm-l] Ajuda em Geometria analítica

2013-09-12 Por tôpico Eduardo Wilner
Pode mas não é necessário,

Como Maldonado mostrou, ao longo do diâmetro ( de equação y/x=m) y/x é 
constante,   
portanto este quociente é o mesmonas extremidades do diâmetro.  




 De: marcone augusto araújo borges marconeborge...@hotmail.com
Para: obm-l@mat.puc-rio.br obm-l@mat.puc-rio.br 
Enviadas: Quinta-feira, 12 de Setembro de 2013 15:07
Assunto: [obm-l] RE: [obm-l] RE: [obm-l] RE: [obm-l] RE: [obm-l] Ajuda em 
Geometria analítica
 


 
Levando em conta que os pontos de intersecção são da forma (x,y) e 
(-x,-y),poderíamos
mostrar,usando a fórmula de distância de um ponto a uma reta,que as distâncias 
de cada um deles às retas tangentes(opostas)são iguais e dai concluir que essas 
retas tangentes são paralelas?




From: joao_maldona...@hotmail.com
To: obm-l@mat.puc-rio.br
Subject: [obm-l] RE: [obm-l] RE: [obm-l] RE: [obm-l] Ajuda em Geometria 
analítica
Date: Thu, 12 Sep 2013 13:18:17 -0300

 
O y/x é constante para os dois pontos de intersecção. Repare que temos 
infinitos m que satisfazem y=mx, mas cada diametro  da elipse é formado por uma 
unica reta (um unico m) que gera dois pontos de intereeccao distintos, porem 
nesses dois pontos o y/x é o mesmo   


From: marconeborge...@hotmail.com
To: obm-l@mat.puc-rio.br
Subject: [obm-l] RE: [obm-l] RE: [obm-l] Ajuda em Geometria analítica
Date: Thu, 12 Sep 2013 12:07:03 +

 
Desculpe,mas por que x/y é constante?




From: joao_maldona...@hotmail.com
To: obm-l@mat.puc-rio.br
Subject: [obm-l] RE: [obm-l] Ajuda em Geometria analítica 
Date: Thu, 12 Sep 2013 02:22:32 -0300

 
Seja a elipse centrada na origem x²/a² + y²/b² = 1
Derivando temos 2xdx/a² + 2ydy/b² = 0, dy/dx = (-x/y) (b²/a²)
Como a reta diametral é da forma y = mx, x/y é constante - dy/dx = constante 
- retas paralelas

[]s
João




From: marconeborge...@hotmail.com
To: obm-l@mat.puc-rio.br
Subject: [obm-l] Ajuda em Geometria analítica
Date: Thu, 12 Sep 2013 02:34:54 +

 
Prove que duas retas tangentes a uma elipse pelos pontos extremos de um 
 
diâmetro são paralelas.
-- 
Esta mensagem foi verificada pelo sistema de antivírus e 
acredita-se estar livre de perigo.
-- 
Esta mensagem foi verificada pelo sistema de antivírus e 
acredita-se estar livre de perigo.
-- 
Esta mensagem foi verificada pelo sistema de antivírus e 
acredita-se estar livre de perigo. 
-- 
Esta mensagem foi verificada pelo sistema de antivírus e 
acredita-se estar livre de perigo.
-- 
Esta mensagem foi verificada pelo sistema de antivírus e 
acredita-se estar livre de perigo. 
-- 
Esta mensagem foi verificada pelo sistema de antiv�rus e
 acredita-se estar livre de perigo.



[obm-l] Re: [obm-l] RE: [obm-l] RE: [obm-l] RE: [obm-l] RE: [obm-l] RE: [obm-l] Ajuda em Geometria analítica

2013-09-12 Por tôpico Eduardo Wilner
Claro que está correto; Seja P1 em uma reta e o pé da perpenciular à outra N1, 
P2 na outra reta 

com N2 pé da perpencidular à ptimeira reta.
P1 N1 P2 N2 representa um retângulo!

[]'s





 De: João Maldonado joao_maldona...@hotmail.com
Para: obm-l@mat.puc-rio.br 
Enviadas: Quinta-feira, 12 de Setembro de 2013 19:33
Assunto: [obm-l] RE: [obm-l] RE: [obm-l] RE: [obm-l] RE: [obm-l] RE: [obm-l] 
Ajuda em Geometria analítica
 


 
Eu nao vejo porque isso estaria certo, se tivermos duas retas, com um ponto em 
cada uma, tal que a distancia de cada um deles à reta oposta é a mesma, não 
quer dizer que as retas sejam paralelas


From: marconeborge...@hotmail.com
To: obm-l@mat.puc-rio.br
Subject: [obm-l] RE: [obm-l] RE: [obm-l] RE: [obm-l] RE: [obm-l] Ajuda em 
Geometria analítica
Date: Thu, 12 Sep 2013 18:07:49 +

 
Levando em conta que os pontos de intersecção são da forma (x,y) e 
(-x,-y),poderíamos
mostrar,usando a fórmula de distância de um ponto a uma reta,que as distâncias 
de cada um deles às retas tangentes(opostas)são iguais e dai concluir que essas 
retas tangentes são paralelas?




From: joao_maldona...@hotmail.com
To: obm-l@mat.puc-rio.br
Subject: [obm-l] RE: [obm-l] RE: [obm-l] RE: [obm-l] Ajuda em Geometria 
analítica
Date: Thu, 12 Sep 2013 13:18:17 -0300

 
O y/x é constante para os dois pontos de intersecção. Repare que temos 
infinitos m que satisfazem y=mx, mas cada diametro  da elipse é formado por uma 
unica reta (um unico m) que gera dois pontos de intereeccao distintos, porem 
nesses dois pontos o y/x é o mesmo   


From: marconeborge...@hotmail.com
To: obm-l@mat.puc-rio.br
Subject: [obm-l] RE: [obm-l] RE: [obm-l] Ajuda em Geometria analítica
Date: Thu, 12 Sep 2013 12:07:03 +

 
Desculpe,mas por que x/y é constante?




From: joao_maldona...@hotmail.com
To: obm-l@mat.puc-rio.br
Subject: [obm-l] RE: [obm-l] Ajuda em Geometria analítica 
Date: Thu, 12 Sep 2013 02:22:32 -0300

 
Seja a elipse centrada na origem x²/a² + y²/b² = 1
Derivando temos 2xdx/a² + 2ydy/b² = 0, dy/dx = (-x/y) (b²/a²)
Como a reta diametral é da forma y = mx, x/y é constante - dy/dx = constante 
- retas paralelas

[]s
João




From: marconeborge...@hotmail.com
To: obm-l@mat.puc-rio.br
Subject: [obm-l] Ajuda em Geometria analítica
Date: Thu, 12 Sep 2013 02:34:54 +

 
Prove que duas retas tangentes a uma elipse pelos pontos extremos de um 
 
diâmetro são paralelas.
-- 
Esta mensagem foi verificada pelo sistema de antivírus e 
acredita-se estar livre de perigo.
-- 
Esta mensagem foi verificada pelo sistema de antivírus e 
acredita-se estar livre de perigo.
-- 
Esta mensagem foi verificada pelo sistema de antivírus e 
acredita-se estar livre de perigo. 
-- 
Esta mensagem foi verificada pelo sistema de antivírus e 
acredita-se estar livre de perigo.
-- 
Esta mensagem foi verificada pelo sistema de antivírus e 
acredita-se estar livre de perigo. 
-- 
Esta mensagem foi verificada pelo sistema de antivírus e 
acredita-se estar livre de perigo. 
-- 
Esta mensagem foi verificada pelo sistema de antiv�rus e
 acredita-se estar livre de perigo.



[obm-l] Re: [obm-l] Re: [obm-l] Re: [obm-l] RE: [obm-l] Re: [obm-l] Teoria dos números

2013-08-30 Por tôpico Eduardo Wilner
Verdade! Obrigado!

[ ]'s





 De: Ralph Teixeira ralp...@gmail.com
Para: obm-l@mat.puc-rio.br 
Enviadas: Sexta-feira, 30 de Agosto de 2013 10:34
Assunto: [obm-l] Re: [obm-l] Re: [obm-l] RE: [obm-l] Re: [obm-l] Teoria dos 
números
 


Acho que voce pensou em 7^x como multiplicacao - ele quer potencia...:-( :-( 

On Aug 29, 2013 9:17 PM, Eduardo Wilner eduardowil...@yahoo.com.br wrote:

Observe que (1 + 3k , 1 + 7k) , k inteiro, satisfaz a equação diofantina

[ ]'s







 De: marcone augusto araújo borges marconeborge...@hotmail.com
Para: obm-l@mat.puc-rio.br obm-l@mat.puc-rio.br 
Enviadas: Quinta-feira, 29 de Agosto de 2013 12:18
Assunto: [obm-l] RE: [obm-l] Re: [obm-l] Teoria dos números
 


7^x modulo 9 dá 1,7 e 4 e 3^y dá quase sempre 0
O que interessa  para 7^x modulo 9 é 4,o que ocorre apenas quando x é da forma 
3.k + 2
Como x tambem é ímpar,só pode ser da forma 6.n + 5,mas...




Date: Thu, 29 Aug 2013 09:21:24 -0300
Subject: [obm-l] Re: [obm-l] Teoria dos números
From: ralp...@gmail.com
To: obm-l@mat.puc-rio.br


Tente agora modulo 9.
On Aug 28, 2013 9:50 PM, marcone augusto araújo borges 
marconeborge...@hotmail.com wrote:

Eu já postei a questão aqui,mas infelizmente não obtive resposta.
Sei que vão aparecendo outras questões interessantes e por isso peço licença 
para reapresentá-la
 
Determine todos os inteiros positivos x,y tais que 7^x - 3^y = 4


Claro que x = 1 e y = 1 satisfaz(desconfio que seja a unica solução)
Eu só consegui concluir que x e y são ímpares,analisando módulo 4.
Desde já agradeço.
 


-- 
Esta mensagem foi verificada pelo sistema de antivírus e 
acredita-se estar livre de perigo. 
-- 
Esta mensagem foi verificada pelo sistema de antivírus e 
acredita-se estar livre de perigo.
-- 
Esta mensagem foi verificada pelo sistema de antivírus e 
acredita-se estar livre de perigo. 


-- 
Esta mensagem foi verificada pelo sistema de antivírus e 
acredita-se estar livre de perigo. 
-- 
Esta mensagem foi verificada pelo sistema de antivírus e 
acredita-se estar livre de perigo. 
-- 
Esta mensagem foi verificada pelo sistema de antiv�rus e
 acredita-se estar livre de perigo.



[obm-l] Re: [obm-l] RE: [obm-l] Re: [obm-l] Teoria dos números

2013-08-29 Por tôpico Eduardo Wilner

Observe que (1 + 3k , 1 + 7k) , k inteiro, satisfaz a equação diofantina

[ ]'s





 De: marcone augusto araújo borges marconeborge...@hotmail.com
Para: obm-l@mat.puc-rio.br obm-l@mat.puc-rio.br 
Enviadas: Quinta-feira, 29 de Agosto de 2013 12:18
Assunto: [obm-l] RE: [obm-l] Re: [obm-l] Teoria dos números
 


 
7^x modulo 9 dá 1,7 e 4 e 3^y dá quase sempre 0
O que interessa  para 7^x modulo 9 é 4,o que ocorre apenas quando x é da forma 
3.k + 2
Como x tambem é ímpar,só pode ser da forma 6.n + 5,mas...




Date: Thu, 29 Aug 2013 09:21:24 -0300
Subject: [obm-l] Re: [obm-l] Teoria dos números
From: ralp...@gmail.com
To: obm-l@mat.puc-rio.br


Tente agora modulo 9.
On Aug 28, 2013 9:50 PM, marcone augusto araújo borges 
marconeborge...@hotmail.com wrote:

Eu já postei a questão aqui,mas infelizmente não obtive resposta.
Sei que vão aparecendo outras questões interessantes e por isso peço licença 
para reapresentá-la
 
Determine todos os inteiros positivos x,y tais que 7^x - 3^y = 4


Claro que x = 1 e y = 1 satisfaz(desconfio que seja a unica solução)
Eu só consegui concluir que x e y são ímpares,analisando módulo 4.
Desde já agradeço.
 


-- 
Esta mensagem foi verificada pelo sistema de antivírus e 
acredita-se estar livre de perigo. 
-- 
Esta mensagem foi verificada pelo sistema de antivírus e 
acredita-se estar livre de perigo.
-- 
Esta mensagem foi verificada pelo sistema de antivírus e 
acredita-se estar livre de perigo. 
-- 
Esta mensagem foi verificada pelo sistema de antiv�rus e
 acredita-se estar livre de perigo.



[obm-l] Re: [obm-l] Teoria dos números

2013-08-28 Por tôpico Eduardo Wilner
Observe que (1 + 3k , 1 + 7k) , k inteiro, satisfaz a equação diofantina

[ ]'s





 De: marcone augusto araújo borges marconeborge...@hotmail.com
Para: obm-l@mat.puc-rio.br obm-l@mat.puc-rio.br 
Enviadas: Quarta-feira, 28 de Agosto de 2013 21:45
Assunto: [obm-l] Teoria dos números
 


 
Eu já postei a questão aqui,mas infelizmente não obtive resposta.
Sei que vão aparecendo outras questões interessantes e por isso peço licença 
para reapresentá-la
 
Determine todos os inteiros positivos x,y tais que 7^x - 3^y = 4

Claro que x = 1 e y = 1 satisfaz(desconfio que seja a unica solução)
Eu só consegui concluir que x e y são ímpares,analisando módulo 4.
Desde já agradeço.
 

-- 
Esta mensagem foi verificada pelo sistema de antivírus e 
acredita-se estar livre de perigo. 
-- 
Esta mensagem foi verificada pelo sistema de antiv�rus e
 acredita-se estar livre de perigo.



Re: [obm-l] Como que faz??

2013-08-25 Por tôpico Eduardo Wilner
De uma ou de outra forma dá bastante equações mesmo! (seriam quatorze?)



[ ]'s




 De: Bernardo Freitas Paulo da Costa bernardo...@gmail.com
Para: obm-l@mat.puc-rio.br 
Enviadas: Sexta-feira, 23 de Agosto de 2013 19:45
Assunto: Re: [obm-l] Como que faz??
 

2013/8/23  douglas.olive...@grupoolimpo.com.br:
 Olá , alguns alunos do ensino médio da instituição onde trabalho me deram
 alguns problemas do site https://brilliant.org/

 PROBLEMA 1: Dada uma função f:R-R tal que f(2x^2 -1)=2(f(x))^2 -1 e f(x) é
 um polinômio de grau 13, sendo assim determine o coeficiente de x^5 de f(x).
Bom, f(x) é dada por 14 coeficientes a_i. A equação que você tem dá um
monte de condições sobre estes coeficientes: para cada x, tem uma
condição.

Por exemplo, x = 0 dá f(-1) = 2f(0)^2 - 1, f(-1) é uma soma alternada,
f(0)^2 é apenas (a_0)^2. Escolhendo um monte de x's, você obtém
bastante equações, e resolve.

Dá pra fazer (um pouco) menos grotesco, porque você pode escrever (a_0
+ a_1 * x + a_2 * x^2 +  + a_13 * x^13)^2, separando por grau.
Duas funções polinomiais em R são iguais se e somente se os
coeficientes forem iguais. Assim, identifique os coeficientes de ambos
os lados, e parta pro abraço.

Pra entender porque isso sempre dá certo, vale a pena lembrar de
Álgebra Linear (também se ensina um pouco desse tipo de intuição em
cursos de Geometria Algébrica classica).

Abraços
-- 
Bernardo Freitas Paulo da Costa

-- 
Esta mensagem foi verificada pelo sistema de antivírus e
acredita-se estar livre de perigo.


=
Instruções para entrar na lista, sair da lista e usar a lista em
http://www.mat.puc-rio.br/~obmlistas/obm-l.html
=
-- 
Esta mensagem foi verificada pelo sistema de antivírus e
 acredita-se estar livre de perigo.



Re: [obm-l] Problemas interessantes

2013-08-24 Por tôpico Eduardo Wilner
Um triângulo  equilátero de lado nse divide em ntriângulos de lado 1 ???!!!

 




 De: Benedito bened...@ufrnet.br
Para: obm-l@mat.puc-rio.br 
Enviadas: Quinta-feira, 22 de Agosto de 2013 4:39
Assunto: [obm-l] Problemas interessantes
 


Segue dois problemas interessantes.
Benedito
 
Problema 1
Um triângulo equilátero de lado 2012 está dividido em 2012  triângulos 
equiláteros menores de lado 1
mediante paralelas ao seus lados. Em cada vértice de um triângulo menor há uma 
formiga. No mesmo instante, 
todas as formigas começam a caminhar com a mesma velocidade pelas retas da 
triangulação.
Ao chegar  a outro vértice giram 60º ou 120º à esquerda ou à direita e  seguem 
movendo-se.
Determinar se é possível que este movimento se desenvolva para sempre sem ter  
nunca duas
formigas em um mesmo vértice de um triângulo menor.
 
Problema 2
Associar aos vértices de um polígono convexo de 33 lados os números inteiros de 
1 a 33, sem repetir, e em seguida, associar aos lados do polígono a soma dos 
números de seus extremos. 
O objetivo é que os números associados aos lados sejam 33 inteiros consecutivos 
ordenados.
-- 
Esta mensagem foi verificada pelo sistema de antivírus e 
acredita-se estar livre de perigo. 
-- 
Esta mensagem foi verificada pelo sistema de antivírus e
 acredita-se estar livre de perigo.



[obm-l] Re: [obm-l] Re: [obm-l] Re: [obm-l] Re: [obm-l] Divisibilidade(congruência)

2013-07-11 Por tôpico Eduardo Wilner
Leia  a mensagem inicial do Marcone Augusto Araujo Borges.
A questão original perdeu-se pelo caminho (parece o jogador que vai driblando e 
e esquece a bola ou a brincadeira do telefone sem fio, de antigamente, 
claro...)  




 De: Bernardo Freitas Paulo da Costa bernardo...@gmail.com
Para: obm-l@mat.puc-rio.br 
Enviadas: Quinta-feira, 11 de Julho de 2013 15:19
Assunto: [obm-l] Re: [obm-l] Re: [obm-l] Re: [obm-l] 
Divisibilidade(congruência)
 

2013/7/11 Eduardo Wilner eduardowil...@yahoo.com.br:

 De: Lucas Prado Melo luca...@dcc.ufba.br
 Para: obm-l@mat.puc-rio.br
 Enviadas: Quinta-feira, 11 de Julho de 2013 6:43
 Assunto: [obm-l] Re: [obm-l] Divisibilidade(congruência)

  2013/7/11 Artur Costa Steiner steinerar...@gmail.com
 
   O Bernardo já mostrou que m + n é múltiplo de 3. Resta mostrar que é 
   também
   múltiplo de 8. Pelo mesmo raciocínio, mn = -1 (mod 8). Para que isto seja
   possível, um dos números m e n tem que ser congruente a 1 módulo 8 e, o
   outro, congruente a -1. Logo, m + n = 1 + (-1) = 0 (mod 8), ou seja,  m + 
   n
   é múltiplo de 8

 m poderia ser 3 e n ser 5.
 3*5 = 15 = 16 - 1 = -1 (mod 8)

 CUIDADO:  nem 3*5+1=16, nem 3+5=8 é divisível por 24

Não, mas o princípio é analisar módulo cada uma das potências de
primos que dividem 24, e o Artur e o Lucas completaram a solução com a
parte mod 8.

 Aliás, a  propriedade vale par qualquer divisor, desde que seja menor que
 pelo menos um entre m e n .

Que propriedade?


-- 
Bernardo Freitas Paulo da Costa

-- 
Esta mensagem foi verificada pelo sistema de antivírus e
acredita-se estar livre de perigo.


=
Instruções para entrar na lista, sair da lista e usar a lista em
http://www.mat.puc-rio.br/~obmlistas/obm-l.html
=
-- 
Esta mensagem foi verificada pelo sistema de antiv�rus e
 acredita-se estar livre de perigo.



Re: [obm-l] Probabilidade - muito interessante...

2013-07-11 Por tôpico Eduardo Wilner
Consideremos o embarque dos 136 passageiros, inclusive você, i.e. excluindo o 
último (consideramos o voo lotado)
Assim que alguém (inclusive você) ocupar o seu lugar ou o do último passageiro 
a embarcar, os passageiros seguintes encontrarão o próprio lugar vago, 
ocupando-o.
Portanto, quando o centésimo trigésimo sétimo passageiro embarcar, encontrará 
vago ou o próprio lugar ou o seu, com probabilidade meio à meio    




 De: Mauricio de Araujo mauricio.de.ara...@gmail.com
Para: obm-l@mat.puc-rio.br 
Enviadas: Quinta-feira, 11 de Julho de 2013 10:16
Assunto: [obm-l] Probabilidade - muito interessante...
 


Recentemente, eu peguei um avião que tinha 137 assentos. Eu gosto sempre de 
ser o primeiro a embarcar e não foi diferente nesta ocasião. Infelizmente, 
assim que eu entrei no avião, percebi que havia perdido o meu cartão de 
embarque e não conseguia me lembrar de qual era o meu assento. Sem saber o que 
fazer, eu escolhi aleatoriamente um assento qualquer e me sentei. Claro que 
havia a probabilidade de 1/137 de eu ter escolhido o assento correto, ou seja, 
aquele que estava marcado no meu cartão de embarque. À medida que os demais 
passageiros embarcavam, cada um se dirigia ao seu assento e sentava-se, a menos 
que o mesmo estivesse ocupado. Neste caso, o passageiro abria mão de sentar-se 
no assento que estava originalmente atribuído a ele (conforme o cartão de 
embarque) e escolhia um outro assento qualquer para se sentar. Percebi que fui 
o único passageiro que perdeu o cartão de embarque.

A questão que se coloca é a seguinte: qual a probabilidade de o último 
passageiro a embarcar encontrar o seu assento desocupado, ou seja, encontrar o 
assento que está no seu cartão de embarque disponível para ele se sentar?

Este problema está explicado no livro Introduction to counting and 
probability do David Patrick e tem uma resposta surpreendente: a probabilidade 
é de 50%...

Para sentir a solução, vale a pena pensar no problema para os casos em que o 
avião tem 2, 3, 4 e 5 assentos... 
-- 

Abraços

oɾnɐɹɐ ǝp oıɔıɹnɐɯ
momentos excepcionais pedem ações excepcionais.

Os cemitérios estão cheios de pessoas insubstituíveis em seus ofícios.
-- 
Esta mensagem foi verificada pelo sistema de antiv�us e 
acredita-se estar livre de perigo. 
-- 
Esta mensagem foi verificada pelo sistema de antiv�rus e
 acredita-se estar livre de perigo.



[obm-l] Re: [obm-l] Divisibilidade(congruência)

2013-07-10 Por tôpico Eduardo Wilner
A formulação não está correta; contra-exemplo : m=3 e n= 9

[ ]'s





 De: marcone augusto araújo borges marconeborge...@hotmail.com
Para: obm-l@mat.puc-rio.br obm-l@mat.puc-rio.br 
Enviadas: Quarta-feira, 10 de Julho de 2013 22:17
Assunto: [obm-l] Divisibilidade(congruência)
 


 
Sejam m e n dois números naturais tais que mn + 1 é multiplo de 24.
Mostre que m + n tambem é múltiplo de 24.

Se possivel,gostaria que alguem resolvesse por congruencia.Obrigado.
-- 
Esta mensagem foi verificada pelo sistema de antivírus e 
acredita-se estar livre de perigo. 
-- 
Esta mensagem foi verificada pelo sistema de antiv�rus e
 acredita-se estar livre de perigo.



[obm-l] Re: [obm-l] Divisibilidade(congruência)

2013-07-10 Por tôpico Eduardo Wilner
Desculpem, desconsiderem ; confundí 24 com 14 (deve ser o sono às duas da 
madruga...)

Boa noite



 


A formulação não está correta; contra-exemplo : m=3 e n= 9

[ ]'s





 De: marcone augusto araújo borges marconeborge...@hotmail.com
Para: obm-l@mat.puc-rio.br obm-l@mat.puc-rio.br 
Enviadas: Quarta-feira, 10 de Julho de 2013 22:17
Assunto: [obm-l] Divisibilidade(congruência)
 


 
Sejam m e n dois números naturais tais que mn + 1 é multiplo de 24.
Mostre que m + n tambem é múltiplo de 24.

Se possivel,gostaria que alguem resolvesse por congruencia.Obrigado.
-- 
Esta mensagem foi verificada pelo sistema de antivírus e 
acredita-se estar livre de perigo. 
-- 
Esta mensagem foi verificada pelo sistema de antiv�rus e
 acredita-se estar livre de perigo.



[obm-l] Prove - desigualdade

2013-05-31 Por tôpico Luís Eduardo Háteras
Sejam x,y,z números reais positivos tais que 1/(1+x) +  1/(1+y) +  1/(1+z) = 2. 
Prove que 8xyz = 1.  

Re: [obm-l] Ajuda

2013-05-12 Por tôpico Eduardo Wilner
Faltam condições...





 De: Marcelo de Moura Costa mat.mo...@gmail.com
Para: obm-l@mat.puc-rio.br 
Enviadas: Domingo, 12 de Maio de 2013 12:11
Assunto: [obm-l] Ajuda
 


Determine as raízes da equação aX² + bX + C = 0  sabendo que 4a - 6b + 9c = 0.

[obm-l] Re: [obm-l] Re: [obm-l] combinatória

2013-05-11 Por tôpico Eduardo Beltrao
Caro Luiz,
Creio que também deve fazer parte deste cômputo os zeros de números tais
quais 103, 1008, 1039, etc.
O número total de zeros será bem maior que os 246 que você achou.

Eduardo


Em 11 de maio de 2013 16:40, Luiz Guilherme Schiefler de Arruda 
lgu...@gmail.com escreveu:

 Considere as seguintes hipóteses:
 I) Cada múltiplo de 10, tem 1algarismo zero (10, 20, 30, ... 2220) -
 totalizando 222 algarismos 0;
 II) Cada múltiplo de 100 tem 2 algarismos zero (100, 200, ... 2200), porém
 1 algarismo zero já foi considerado na hipótese anterior - totalizando 22
 algarismos 0;
 III) Cada múltiplo de 1000 tem 3 algarismos zero (1000 e 2000), porém 2
 algarismos zero já foram considerados nos múltiplos de 10 e 100 -
 totalizando 2 algarismos 0;

 Somando os três totais temos: 222 + 22 + 2 = 246 números 0
 -
 Luiz Guilherme

 Em 02/05/2013, às 15:34, Luciane Barbosa lubarbo...@aol.com escreveu:


 peessoal, estou quebrando a cabeça com esse problema mas tá complicado...

 Escrevendo-se os números inteiros de 1 até , quantas vezes o algarismo
 0 aparece?

 bjs, Lu.





Re: [obm-l] Ajuda

2013-05-05 Por tôpico Eduardo Wilner
No texto inicial, a gritante interrogação, se refere ao que? Ao lado?

[ ] s  




 De: Marcelo de Moura Costa mat.mo...@gmail.com
Para: 
Enviadas: Domingo, 5 de Maio de 2013 5:42
Assunto: [obm-l] Ajuda
 


Tenho certeza de que alguém da lista já se deparou com esse problema e sua 
solução:

Um ponto interno de um triângulo equilátero dista 5cm, 7cm e 8cm dos vértices 
do triângulo.?

Solução:

3(p^4 + q^4 + t^4 + a^4) = (p^2 + q^2 + t^2 + a^2)^2.

p = 5
q = 7
t = 8

a=lado do triângulo equilátero. 

Alguém já viu a demonstração ou conhece?
Agradeceria a informação.

Abraços e boa semana.

Marcelo

[obm-l] Re: [obm-l] Sistema de Três Equações com Quadrados

2013-05-05 Por tôpico Eduardo Wilner
Deve haver algum engano: sistema de três equações (parecem L.I.) à duas 
incógnitas?




 De: terence thirteen peterdirich...@gmail.com
Para: obm-l obm-l@mat.puc-rio.br 
Enviadas: Quarta-feira, 1 de Maio de 2013 21:02
Assunto: [obm-l] Sistema de Três Equações com Quadrados
 



Resolva o sistema abaixo:

3(S-l)^2+D^2=3^2
3S^2+(l-D)^2=4^2
3S^2+(l+D)^2=5^2



(Espero que minha formulação esteja correta...)


-- 
/**/
神が祝福

Torres 

[obm-l] Re: [obm-l] Polígono regular inscrito

2013-05-02 Por tôpico Eduardo Wilner
Os fantásticos números complxos resolvem.

Vejamos, como ilustração, o triângulo ( e um pouco do quadrado) que vc. 
resolveu geométricamente

Sejaz = e^(ib)  , 0 =b2pi,  representando os pontos da circunferência com 
centro na origem do plano complexo(Argand-Gauss) .

As raizes de terceira ordem de 1, i.e., da equação z^3=1, representam os 
vértices do triângulo inscrito:

 e^(i3b) =1   = 3b=j.2.pi = b=j.2.pi/3 , j= {0,1 ,2}; assim, os vértices do 
triângulo localizam-se em   


z_0=1, no semi-eixo real positivo, (seja o vértice V_0) , z_1=e^(i2.pi/3)  e  
z_2=e^(i4.pi/3) .

Poderiamos também partir da equação  z^3 - 1 =0 (I)  , e sabendo que uma 
solução é z=z_0=1, dividi-la por z - 1 obtendo, z^2 + z + 1=0 , que reproduz as 
mesmas raizes acima, z_1 e z_2  (z_2 apareceria como e^(-i2.pi/3)=e^(i4.pi/3) ).

Para as cordas, sem perda de generalidade, escolhemos o vértice V_0 como o 
comum à elas e definimos como w = z  - z_0 =z -1 as cordas vetoriais. Fazendo 
a mudança de variável na equação (I), 
   (w+1)^3  - 1 =0  = w^3 + 3w^2 + 3w  = 0 , que dividida por w=w_0= 0 (a 
corda V_0V_0, que gracinha...) resultando em w^2 +3w + 3 =0 cujas raizes são as 
de z subtraidas de 1. Mas , no caso, o que importa é que o termo indepente 
fornece o produto das raizes, 3. ( observe  que parao comprimento das cordas 
teriamos que trocar w por  |w| mas como, no produto há compensação de sinais, 
não é necessário.    


 Para o quadrado mantendo z=z_0=1 e dividindo a equação z^4 - 1= 0 (II)por z-1, 
obtemos a equação
z^3+z^2+z+1=0 , cujas raizesrepresentam os outros 3 vértices (os dois do eixo 
imaginário e o do semi=eixo real positivo. Fazendo a mudança de variável, z = w 
+ 1 em (II),
  (w+1)^4 - 1 =0  = w^4 + 4w^3+6w^2+4w=0, que dividida por w fornece  


   w^3+4w^2+6w+4 = 0, termo independente , produto das raizes, 4.

Genéricamente, para o polígono de n lados, a equação z^n - 1 = 0 (III), em 
termos de w,

(w+1)^n  - 1 = w^n + n w^(n-1) + ...+ (n , j) w^(n-j) +...n=0 , onde (n , j) 
são os coeficientes binomiais, mostra que o termo independente , produto das 
raizes, é n.

[ ]'s  


    




 De: Martins Rama martin...@pop.com.br
Para: obm-l@mat.puc-rio.br 
Enviadas: Quarta-feira, 1 de Maio de 2013 3:53
Assunto: [obm-l] Polígono regular inscrito
 

Caros amigos da lista...
A afirmação abaixo é verdadeira? Como prová-la? Indução, talvez?

Para um polígono regular convexo de n vértices V1, V2, ...,Vn, inscrito
num círculo de raio unitário, qual o valor do produto das medidas das
(n-1) cordas traçadas de um vértice, por exemplo, V1?

P = V1V2 x V1V3 x ... x V1Vn = ?

Sei que para o:
- triângulo equilátero, temos: (raiz de 3)x(raiz de 3) = 3
- quadrado, temos: (raiz de 2)x(raiz de 2)x2 = 4
- hexágono regular, temos: 1x(raiz de 3)x2x(raiz de 3)x1 = 6

É possível generalizar a solução e encontrar a resposta n para todos os
polígonos regulares?

Abraços,
Martins Rama.

=
Instruções para entrar na lista, sair da lista e usar a lista em
http://www.mat.puc-rio.br/~obmlistas/obm-l.html
=

Re:[obm-l]Questoes

2012-11-28 Por tôpico Eduardo Wilner
Me perdí no novo formato. Existe algum botão para postar resposta? E um novo 
subject

Assim, estou tentando à moda antiga = por e-mail...

Fabio Silva pode dividir o quadrilátero em dois triângulos, por exemplo, de 
mesma base 4 e alturas  x.sen 30º=x/2 e  (10-x).sen 30º=(10 - x)/2.

Assim a àrea será

 (1/2).4.(x+10-x)/2= 10.

[ ]'s


Re: [obm-l] Ajuda em geometria

2012-11-04 Por tôpico Eduardo Wilner


Podemos considerar um dos quadriláteros como um quadrado de vértices Qi e 
pontos médios Mi i(1 2 3 4) e o outro convexo qualquer de vértices Gi, que a 
transitividade garante a generalidade.

Devido a convexidade teremos vértices opostos, sejam os de i impar, do quadrado 
no interior ao genérico, seja i=1, e o de i=3, ao contrário: o vértice do 
genérico no interior do quadrado. Os outros vértices, de i par são, de cada 
quadirlátero externo ao outro.

Cada triãngulo QiMiGi é congruente ao Q(i+1)MiG(i+1), até i igual a3 (lógico) e 
completa com Q4M4G4 congruente a Q1M4G1.

Assim, a área do triângulo de i=1, excesso do genérico em relação ao quadrado é 
compensado pelo seu congruente, excesso do quadrado em relação ao gnérico , com 
exceção de um triângulo (seja Q2G2E2) externo à ambos, mas que compensa na 
comparação do par seguinte , i.e., de i=2 com seu congruente. 

E assim repete-se num ciclo, completando as áreas.

[ ]'s 


[obm-l] Re:[obm-l] FW: Tentei e não consegui(geometria)

2012-10-25 Por tôpico Eduardo Wilner
Pela semelhança dos triangulos ABE (base AB=a, altura h2) e CDE(base CD=b, 
altura h3) ,
  (a/h2) = (b/h3) = k.
Assim,  A2 = a*h2/2 = (k/2)(h2)^2  = h2 = \sqrt (2*A2/k) (I)

Analogamente  A3 = (k/2)(h3)^2   =  h3 = \sqrt(2*A3/k)   (II)

 A1 = (h/2)(a+b) = (h/2)k(h2+h3) = (k/2)h^2 (pois  (IV) h2+h3=h) = h 
=\sqrt(2*A1/k) (III)

Substituindo  (I), (II) e (III) em (IV) e simplificando, obtemos a relaçao 
proposta.

[ ]'s


[obm-l] Re:[obm-l] Geometria(Construção(2))

2012-09-11 Por tôpico Eduardo Wilner
Parece que há uma inversão na posição dos pontos, não?

[ ]'s



[obm-l] Re: [obm-l] Re: [obm-l] Socorro em geometria (construçã o)

2012-09-09 Por tôpico Eduardo Wilner
Soh para ficar bem claro: eh por C passe uma paralela a AT.

[ ]'s

--- Em sáb, 8/9/12, Julio César Saldaña saldana...@pucp.edu.pe escreveu:

De: Julio César Saldaña saldana...@pucp.edu.pe
Assunto: [obm-l] Re: [obm-l] Socorro em  geometria (construçã o)
Para: obm-l@mat.puc-rio.br
Data: Sábado, 8 de Setembro de 2012, 8:51



Contrói o triângulo ACT com lados AC=p, CT=q e AT=s.

Por T passe uma paralela a AT. Com centro em A desenhe um arco de raio a. Esse
arco vai cortar à paralela (tem 2 soluções). Chame de D a ese ponto de corte.
Por D traçe uma paralea a CT, o ponto de corte entre essa paralea e AT será o
ponto B do trapecio ABCD procurado.

Julio Saldaña


-- Mensaje original ---
De : obm-l@mat.puc-rio.br
Para : obm-l@mat.puc-rio.br
Fecha : Sat, 8 Sep 2012 03:09:59 +
Asunto : [obm-l] Socorro em  geometria (construçã o)




Construir o trapézio ABCD conhecendo a soma das bases AB +CD = s,as diagonais
AC = p e BD = q e o lado AD = a.Justifique.                        

__
Si desea recibir, semanalmente, el Boletín Electrónico de la PUCP, ingrese a:
http://www.pucp.edu.pe/puntoedu/suscribete/

=
Instruções para entrar na lista, sair da lista e usar a lista em
http://www.mat.puc-rio.br/~obmlistas/obm-l.html
=


[obm-l] Re: [obm-l] Ajuda e orientações

2012-09-04 Por tôpico Eduardo Wilner
Por isso o enunciado coloca se duas parábolas,,,

Uma maneira de provar, usando GA seria escolher, spg, uma parábola com a 
diretriz coincidente com Ox, de foco (a,b) e a outra com a diretriz 
coinicidindo com Oy e foco (c,d), sendo (x,y) qualquer dos quatro pontos de 
intersecção (claro que os focos são tais que a mencionada
 condição ocorra).  

Usando a condição de definição (mesma distância ao foco e à diretriz) para a a 
primeira parábola, temos

    y^2 = (x-a)^2+(y-b)^2   =   x^2 - 2.a.x - 2.b.y = - b^2 - a^2  
(I)

e para a segunda 

                x^2= (x-c)^2+(y-d)^2 =   y^2 -2.d.y - 2.c.x = - c^2 - d^2 
(II)

Somando membro a membro, (I) + (II)

               [x - (a+c)]^2 - (a+c)^2  +  [y - (b+d)]^2 - (b+d)^2 =  - b^2 - 
a^2 - c^2 - d^2

 ou    

   [x - (a+c)]^2  +  [y - (b+d)]^2  =  2(a.c+b.d)

    que relaciona claramente a posição dos focos com o  raio e o centro da 
circunferencia. 


--- Em dom, 2/9/12, Marcelo de Moura Costa mat.mo...@gmail.com escreveu:

De:
 Marcelo de Moura Costa mat.mo...@gmail.com
Assunto: [obm-l] Ajuda e orientações
Para: obm-l@mat.puc-rio.br
Data: Domingo, 2 de Setembro de 2012, 17:27

Foi-me apresentado o seguinte problema:

Mostre que se duas
 parábolas, com retas focais perpendiculares entre si, se
intersectam em quatro pontos, então estes pontos pertencem a um círculo.

O problema começa em que o fato das retas focais serem perpendiculares não 
garante


que haverá 4 pontos de intersecção entre as parábolas, é necessário pelo menos 
que os focos de ambas 
encontrem-se no mesmo quadrante formado pelas perpendiculares e a uma 
determinada distância.
Ou eu estou enganado? Gostaria muito de uma orientação quanto a esse problema.




[obm-l] Re: [obm-l] Ajuda e orientações

2012-09-04 Por tôpico Eduardo Wilner
Em tempo...(?não dá para editar...) Estou procurando uma demonstração mais 
elegante
i.e., sem GA.

[ ]'s


[obm-l] Re:[obm-l] Re: [obm-l] Geometria Plana - Triângulo

2012-08-31 Por tôpico Eduardo Wilner
Carlos Vitor, poderia explicar por que o quadrilatero ACHE eh ciclico?

Vc. estah considerando EH paralelo a AC? Por que?
 
[ ]'s



[obm-l] Re: [obm-l] problema da divisão

2012-08-14 Por tôpico Eduardo Wilner
Me parece que estipulando que aquele que divide é o ultimo a pegar sua parte, 
resolve.

Ou não é este o espírito da questão? 


[ ]'s


Re: RE: [obm-l] Geometria espacial

2012-08-05 Por tôpico Eduardo Wilner
Acho que estamos falando aqui sobre o caso em que o raio das esferaas eh 
máximo, i.e., cada face do tetraedro tangencia tres das esferas.
Assim, Maldonado, seu tetraedro estah muito subdimensionado; vc. soma um r ah 
altura do tetraedro interno na base, OK, mas um r no vertice(?) não estah OK.
Acho
 mais facil considerar que a aresta eh proporcional ao inraio , um 
quarto da altura,  e levar em conta que o inraio do externo eh igual ao do 
interno mais um raio das esferinhas, chegando sem problemas ah 

 a = 2r(1+sqrt6).

Que
 tal considerar o outro extremo; esferas com o minimo raio, i.e. , cada 
face do tetraedro tangenciando apenas uma das esferas?

[ ]'s

Re:Re: [obm-l] Geometria Espacial PIRAMIDE

2012-07-26 Por tôpico Eduardo Wilner
É verdade; eu assumí a reta r passando pelo ponto O...

[ ]'s


Re: [obm-l] Geometria Espacial PIRAMIDE

2012-07-21 Por tôpico Eduardo Wilner
Parece haver algum engano, ou eu não entendí o enunciado

Podemos construir um corte vertical da pirâmide como um triângulo retângulo 
com um cateto sendo a metade da aresta, a/2, a hipotenusa como a altura do 
triângulo equilátero, da face lateral, (a/2) 3^(1/2), portanto o outro cateto, 
altura da pirâmide, (a/2)2^(1/2).

Assim, a distância pedida é a altura, d , desse triângulo 

 d = (a/2)(a/2)2^(1/2)/[(a/2)3^(1/2) = (a/6)6^(1/2)

[ ]'s 


Re:[obm-l] Probleminha

2012-07-18 Por tôpico Eduardo Wilner



Seja X o volume do tonel e x o volume da caneca.
Na primeira operação restou X-x de vinho e x de água.

Admitindo que o cliente agitou bem antes de usar a segunda dose, foi retirado 
(x/X)x de água e reposto x, logo  a quantidade final de água será 2x-(x^2)/X = 
X/2.

Resolvendo, a solução (menor que X) é x = X (2-sqrt2)/2.

Se for dirigir, não beba!

[ ]'s 


[obm-l] Re:[obm-l] A função e^x

2012-06-11 Por tôpico Eduardo Wilner
No item 1) a e b são reais?


[obm-l] RE:[obm-l] Eu não entendo este problema

2012-05-13 Por tôpico Eduardo Wilner







Parece que drechum é estrangeiro... de onde ?Neste 
problema, se bem o entendí ( bem quer dizer médio...) o raio da circunferência 
é igual aos lados do triângulo e do quadrado, que parece ser 2 (?)
[ ]'s


drechum

Sun, 13 May 2012 14:07:02 -0700








ABC é um triângulo eqüilateral; BCDE é um quadrado 2 lateral construído 
exteriormente ao triângulo. Os vértices A, D e E que eles pertencem à mesma 
circunferência. Acha o valor do rádio da circunferência.

  1   2   3   4   5   6   7   8   9   10   >